You are on page 1of 129

GENERAL PHARMACOLOGY

1. Lipid soluble drugs with adequate concentration gradient and lipid: water coefficient depend for their
absorption on:
a) Endocytosis
b) Filtration
c) Active transport
d) Facilitated diffusion
e) Simple diffusion
2. The set of changes that the drugs undergo on entery into the body are referred to as:
a) Absorption
b) First pass effect
c) Distribution
d) Excretion
e) Pharmacokinetics
3. The set of changes undergone by the body by a drug is referred to as:
a) Oxidation
b) Hydrolysis
c) Distribution
d) Pharmacokinetics
e) Pharmacodynamics
4. The large sized particles permeate into the body by:
a) Simple diffusion
b) Facilitated diffusion
c) Active transport
d) Endocytosis
e) Filtration
5. Which of the following drug property favours the drug transport?
a) Hydrophilic nature
b) Low diffusion gradient
c) Ionization
d) Large molecular size
e) Non polar nature
6. Regarding the administration-distribution relation ship the correct statement is:
a) Small volume of distribution favours rapid BBB distribution
b) Drug entering highly vascular compartment remains there and redistributes slowly elsewhere.
c) Fat rich compartment draws maximum drug due to its more uptake and binding
d) Drugs with large volume of distribution distribute to all the compartments
e) Muscle proteins draws the drugs with high plasma protein binding capacity
7. Sulfonamide induced warfarin bleeding in a patient taking alcohol can be avoided by:
a) By decreasing the dose of sulfonamide while drinking alcohol
b) By decreasing the dose of warfarin while drinking alcohol
c) By decreasing the dose of warfarin while taking sulfonamide concurrently
d) By decreasing the dose of sulfonamide while taking warfarin concurrently
e) By discontinuing warfarin while taking sulfonamide
8. Sulfonamide induced warfarin bleeding in a patient taking alcohol is caused by:
a) Sulfonamide prevents the renal secretion of warfarin
b) Sulfonamides competes with warfarin at its receptor
c) Bleeding is also an adverse effect of sulfonamides
d) Sulfonamides increase the growth of intestinal bacteria that are respobsible for degrading warfarin
e) Sulfonamides inhibit the metabolism of warfarin and increase its level in blood
9. A patient with proper therapeutic level of a drug develops toxicity . What can be the possible cause?
a) Strenous exercise increasing the muscle mass and holding a major portion of drug from the blood
b) An other drug inhibiting the hepatic metabolism of the drug
c) Another drug increasing the protein binding of the drug
d) Another drug increasing the hepatic firtst pass effect of the drug
e) Taking the drug with food
10. The phenomenon where the increased drug concentration is associated with increased metabolism and
excretion is called:
a) High elimination half life
b) Zero order kinetics
c) First order kinetics
d) Saturation kinetics
e) High extraction ratio
11. In a patient with hypertension and asthma the contraindication and justification of a beta blocker is
explained as follows :
a) Propranolol being a nonselective blocker can aggravate asthma
b) Propranolol being a nonselective blocker has lesser efficacy as an antihypertensive
c) Propranolol being a nonselective blocker would not fully control asthmatic attacks
d) Atenolol being a cardio selective drug may trigger an asthmatic attack
e) Attenolol might fail in controlling asthmatic attacks
12. Regarding the CNS side effects of propranolol versus atenolol which of the following statement is correct?
a) Propranolol reaches the CNS in therapeutic concentration more readily due to its large volume of
distribution
b) Propranolol will more readily cause its CNS side effects as it is more lipophilic and can readily cross
BBB
c) Propranolol will more readily be trapped in the fat compartment due to its large volume of
distribution and will cause cause its CNS side effects
d) Atenolol due to its larger volume of distribution is more liable to cause its CNS side effects
e) Atenolol due to its smaller volume of distribution will not be able to reach CNS in therapeutic
concentrations
13. In age related impairment of liver function the choice of dose and beta blocker is :
a) Reduced dose of propranolol because it is more highly metabolized by the liver and may cause drug
accumulation and adverse effects
b) Reduced dose of propranolol because it is extensively metabolized by the liver and causes hepatic
failure
c) Increased dose of propranolol for its therapeutic level because it is less highly metabolized by the liver
d) Increased dose ofatenolol for its therapeutic level because it is less highly metabolized by the liver
e) Reduced dose of atenolol because it is extensively metabolized by the liver and may cause drug
accumulation and adverse effects
14. Regarding the plasma protein binding of propranolol versus atenolol the correct guideline for a thiazide
prescription is:
a) Propranolol being more highly protein bound needs lower dose
b) Propranolol being more highly protein bound is more likely to compete with thiazide for binding
c) Propranolol being less highly protein bound is less likely to compete with thiazide for binding
d) Atenolol being less highly protein bound is less likely to compete with thiazide for binding
e) Atenolol being less highly protein bound has lesser ability to reach its receptors
15.Which of the following routes of administration partially avoids the first pass effect?
a) Sub lingual

b) Rectal

c) Transdermal

d) Oral

e) Subcutaneous

16.Which of the following route is the serving for the rapid action of glyceryl trinitrate in a patient of angina

with severe chest pain?

a) Intramuscular

b) Oral

c) Transdermal

d) Subcutaneous

e) Sublingual

17: The determining factor for a drug given by infusion depends on which of the following factors for
achieving steady state concentration:
a) Rate of infusion
b) Distribution
c) Volume of distribution
d) Half life
e) Bioavailability

18.The volume of distribution of drugs will be greater if the drug:

a) Is more ionized inside cells than in plasma

b) Is administered very rapidly

c) Highly ionized in plasma

d) Has poor lipid solubility

e) Has a high molecular weight

19. Bioavailability of a drug is reduced by:


a) Metabolic degradation
b) Rapid excretion
c) First pass effect
d) Enzyme induction
e) Pharmacodynamics

20. Ethanol is eliminated by:


a) First pass effect
b) Rapid distribution
c) Zero order kinetics
d) First order elimination
e) Urinary excretion
21. The primary site for oxidation reduction reactions is:
a) Nuclei of liver cells
b) Cytosol
c) Endoplasmic reticulum
d) Intestinal epithelium
e) Plasma membrane of liver cells
22. The primary site for conjugation reactions and hydrolysis is the :
a) Plasma membrane of liver cells
b) Cytosol
c) Cell nucleus
d) Miochondria
e) Golgi bodies
23. Some drugs after absorption go to the liver and are metabolized thus decreasing their biopavailability. The
phenomenon is called:
a) Enzyme induction
b) Xenobiosis
c) Portal hypertension
d) First pass effect
e) Conjugation
24. In HIV positive patients with tuberculosis and using saquinavir, the use of Isoniazid becomes a matter
of concern with people having certain ethnic background because:
a) In slow acetylators isoniazid is not absorbed properly
b) Slow acetylators are at risk of isoniazid toxicity
c) Slow acetylators develop supra therapeutic saquinavir concentrations
d) Fast acetylators have CYP 450 3A4 enzyme induction
e) Fast acetylators are at risk for supratherapeutic isoniazid concentrations
25. In HIV patient having tuberculosis and using rifampin it is important to consider that:
a) Rifampin diverts protease inhibitors to the kidney for rapid elimination
b) Rifampin competes with protease inhibitors for binding to P450 3A4
c) Rifampin inhibits the activity of MDRI transporter
d) Rifampin induces the production of glucoronic acid
e) Rifampin induces the activity of CYP 450 3A4 enzyme
26. In HIV patient having tuberculosis and using saquinavir it is important to consider that:
a) Saquinavir prevents the renal elimination of anti tuberculous drugs
b) Drinking grapefruit juice will enhance the metabolism of saquinavir
c) Antituberculous drug enhance the metabolism of saquinavir
d) In HIV Patient the G.I absorption of antituberculous drugs is blocked by multi drug resistant protein
1(MDR1)
e) Development of oral thrush prevents the absorption of antituberculous drugs
27.Periodic blood samples taken before and repeatedly after the administration of a drug plotted as log of
concentration versus time and showing slope of the curve depicts information regarding:
a) Bioavailability
b) Area under the curve (AUC)
c) Elimination constant
d) Protein binding
e) Volume of distribution
28. The number of drug receptor complexes depends upon:
a) Dose of the drug
b) Efficacy of the drug
c) Half life of the drug
d) Affinity of the receptors for the drug
e) Bioavailability of the drug
29.Which characteristic of the phase II biotransformation reactions is valid:
a) They always transform the prodrugs into active compounds
b) They create an active centre for more conjugations
c) They are induced by repeated doses
d) They render the drug more lipid soluble
e) They almost always produce water soluble products

30.A non competitive antagonist is one which:

a) Binds to the receptor at the site of agonist binding


b) Causes alteration in the mechanism of action of the agonist
c) Shifts the dose response curve to the right
d) Causes potency alteration of the agonist
e) Decreases the magnitude of response to an agonist
31. The absorption of a drug from the intestines can be increased by decreasing the pH of the intestinal lumen
when the drug given is:
a) a weak acid
b) a weak base
c) a neutral compound
d) highly ionized compound
e) organic in nature
32.Weakly acidic drugs are rapidly absorbed form:

a) Stomach

b) Small intestine

c) Large intestine

d) All of them

e) None of them

33: Which of the following statements applies to the zero order kinetics of elimination:
a) Rate of elimination is dependant on plasma concentration
b) The fraction of the drug eliminated per unit time remains fixed
c) Multiple doses will cause achievement of the steady state concentration in approximately 4 to 5
biological half lives.
d) There is no difference in the half-life, no matter now much dose is administered
e) The dose of the drug affects the biological half life

34: Displacement of a drug from its plasma protein binding will lead to:
a) Decrease in the plasma level of the drug
b) Decrease in the volume of distribution
c) Decrease in the metabolism of the drug
d) Decrease in the excretion of the drug
e) Increase in the tissue level of the drug
35. Solubility of a drug in lipid medium of a well distributed drug is specially related to its:
a) Transport by aqueous diffusion
b) Transport by exocytosis
c) Carrier mediated transport
d) Transport by diffusion through the membranes
e) Transport by pinocytosis
36. Aspirin toxicity can be treated by:
a) Administration of acetic acid
b) Administration of tartaric acid
c) Administration of an organic base
d) Administration of ammonium chloride
e) Administration of sodium bicarbonate
37.The reciprocal of dissociation constant (KD) is:
a) EC50
b) Potency
c) Efficacy
d) Affinity
e) Intrinsic activity
38. The dose response relationship in vitro is expressed best by:
a) Hyperbolic curve
b) Parabolic curve
c) Linear curve
d) Gaussian normal curve
e) Sigmoidal curve
39. A competitive antagonist will affect the concentration response curve as:
a) It will decrease the EC50 and will also decrease the minimum effect
b) It will decrease the EC50 but will not decrease the maximum effect
c) It will increase the EC50 but will not change the maximum effect
d) It will increase the EC50 and decrease the maximum effect
e) It will not change EC50 but will decrease the maximum effect

40. Using protamine as an antidote of heparin is an example of:


a) Physiological antagonism
b) Allolsteric antagonism
c) Competitive antagonism
d) Irreversible antagonism
e) Chemical antagonism
41. Using adrenaline to antagonize the effect of histamine in anaphylaxis is an example of:
a) Chemical antagonism
b) Competitive antagonism
c) Physiological antagonism
d) Irreversible antagonism
e) Allosteric antagonism
42. The ratio between the toxic dose to the desired dose is reflected by the term:
a) Median effective dose
b) Median toxic dose
c) Median lethal dose
d) Pharmacological efficacy
e) Therapeutic index
43.An infrequent and unusual drug response is termed as:
a) Tolerance
b) Tachyphylaxis
c) Hyperreactive response
d) Idiosyncratic response
e) Hypo reactive response
44. Which of the following is related to the quantal dose response curve?
a) Relaxation of isolated intestinal smooth muscle
b) Increasing natriuresis in CCF
c) Cardiac arrhythmia reverting back to normal rhythm in a population
d) Increasing force of contraction of a failing heart
e) Lowering of blood pressure in an hypertensive patient

45. Adding or unmasking of a functional group in biotransformation is called:


a) Synthetic reaction
b) Conugation reaction
c) Glucoronidation
d) Phase I reaction
e) Phase II reaction
46 Phase II biotransformation reactions include:
a) Oxidation
b) Deamination
c) Hydroxylation
d) O-dealkylation
e) Acetylation
47. Cytochrome P450 enzymes are located in:
a) Golgi apparatus
b) Nucleus
c) Cytoplasm
d) Smooth endoplasmic reticulum
e) Rough endoplasmic reticulum
48. Rifampin alters the response of contraceptives by:
a) Inducing the drug metabolizing enzymes
b) Interfering with the follicile stimulating hormone and lateinizing hormone
c) Inhibiting ovulation by its action on avaries
d) It has a direct effect on endometrium
e) It inhibits the enzyme that metabolizes the oral contraceptives.
49.Bioavailabilty is:
a) The amount of absorbed drug
b) The amount that goes to the blood in any form
c) Reduced when a drug is eliminated by forced dieresis
d) Amount of drug that can produce the desired effect
e) The concentration of a drug reaching blood in unchanged form
50. If concentration vs time curve is plotted ,Bioavailability:
a) Is represented by the magnitude of dose
b) Is shown by the time factor
c) Is shown by a linear curve
d) Is shown by the the area under the curve
e) Is shown by a sigmoid shaped curve

51. High Bioavailability of a drug depends upon the:


a) Hydrophilicity of drug
b) Lipophilicity of drug
c) Both hydrophillicity and lipophilicity of drug
d) Neither hydrophhilicity nor lipophilicity
e) High hydrophilicity and low lipophilicity

52. Extraction ratio of a drug is:


a) Directly proportional to the hepatic blood flow
b) Directly proportinal to the liver clearance of a drug
c) Inversely proportional to the liver clearance
d) The concentration of drug extracted for effect
e) The proportion of a drug extracted as activeprinciple

53. In the drugs administered by rectal route:


a) Inferior most part of rectum allows maximum absorption andbioavailability
b) Inferior most part of rectum allows least if at all absorption
c) Middle part of rectum allows maximum absorption
d) Middle part of rectum allows minimum absorption
e) Superior part of rectum allows maximum absorption

54. All of the following statements concerning the blood brain barrier and the passage of drugs from the

systemic circulation into the cerebrospinal fluid are true EXCEPT:

a) Ionized drugs are more likely to cross into the CSF than un-ionized drugs

b) The higher the lipid solubility of a drug, the more likely it will cross into the CSF.

c) Inflammation of the meninges improves the likelihood that drugs will cross the blood brain

barrier as compared to the non-inflamed state

d) P glycoprotein serves to pump drugs back into the systemic circulation from endothelial cells

lining the blood brain barrier.

e) None of them

55. We start intravenous infusion of a drug using a pump that ensures that the rate of drug delivery is

constant over time. Which of the following factors determines how long it will take for the drug to reach a

steady state concentration (CSS) in the blood?

a) Apparent volume of distribution

b) Bioavailability

c) Clearance

d) Half-life

e) Infusion rate (mg of drug/min)

56. Which of the following drug crosses the blood brain barrier?

a) Domperidone

b) Propranolol

c) Suxamethonium

d) Edrophonium

e) Dopamine

57. Apparent volume of distribution is:


a) The volume of drug when dissolvedin total body water
b) Apparent plasma volume in the intravascular compartment
c)Total volume of drug when dissolved in extra cellular fluid
d) Ratio between intracellular and extracellular fluid compartments
e) Ratio of the amount of the drug in the body to its concentration in the plasma

58. The total volume of extracellular fluid is:


a) 42 litres
b) 24 litres
c) 14 litres
d) 28 litres
e) 4 litres

59. Total volume of a drug distributed and confined to the plasma is:
a) 4 litres
b) 1 4 litres
c) 24 litres
d) 28 litres
e) 42 litres
60. T he total body water volume is:
a) 14 litres
b) 42 litres
c) 28 litres
d) 4litres
e) 24 litres

61. What is the volume of distribution of a drug whose concentration in the plasma2 µg/ml and total amount
of drug in the body is 90mg?
a) 450L

b) 45L

c) 90L

d) 9L

e) 900L

62. Dialysis is useful when:


a) The drug is leaves the plasma compartment
b) The drug is tightly bound to the plasma proteins
c) The drug is mainly in the interstitial fluid
d) The drug has gone to the extracellular fluid
e) The drug is confined to the intravascular compartment
63. In biotransformation:
a) Polar and water soluble substances are rendered nonpolar and lipid soluble.

b) Non polar and water soluble substances rendered polar and lipid soluble.

c) Polar and lipid soluble substances are rendered nonpolar and water soluble.

d) Polar and lipid soluble substances are rendered nonpolar and lipid soluble.

e) Non polar and lipid soluble substances are rendered polar and water soluble.

64. The addition of glucoronic acid to a drug :


a) Decreases its water solubility

b) Involves cytochrome P450

c) Is an example of phase I reactions

d) Occurs at the same rate in adults and newborn

e) Usually leads to inactivation of drugs

65. A drug undergoes extensive entero-hepatic circulation. What is the effect of this on its

pharmacokinetics?

a) Decrease bioavailability of the drug.

b) Prolong the duration of action of drug.

c) Promotes rapid excretion of the drug in feces

d) Results in increase dose requirement

e) There will be more plasma protein binding

66. Suxamethonium is metabolized in:


a) Liver

b) GIT

c) Plasma

d) Lungs

e) Kidneys

67. All of the following drugs undergo first pass effect:


a. Morphine, propranolol, benzodiazepine, glycerol trinitrate
b. Chlorpromazine, propranolol,glycerol trinitrate,nortriptyline.
c. Propranolol, warfarin ,morphine,glycerol trynitrate.
d. Nortriptyline, morphine,aspirin, glycerol trynitrate
e. Morphine ,warfarin, Chlorpromazine, glycerol trinitrate.
68. All of the following are Non microsomal enzymes Except:
a) MAO

b) Insulinase

c) CYP 450

d) Aldehyde dehydrogenase

e) Pseudocholinestrase

69. Phase –II reactions are mostly:


a) Oxidation reactions

b) Reduction reactions

c) Hydrolysis reactions

d) Acetylation reactions

e) Conjugation reactions

70. Tubocurarine antagonizes acetyle choline by acting:

a) Chemically

b) Physiologically
c) Competitively

d) Non-Competitively

e) Irreversibly

71. Verapamil antagonizes the calcium ions by:

a) Acting competitively

b) Acting Physically

c) Acting non competitively

d) Acting Physiologically

e) Acting on the same receptors

72. In noncompetitive antagonism the curves are:

a) Parrallel

b) Non- parallel

c) Shifted to the left

d) Increased in slopes

e) Increased in response

73. Surmountable block involves:

a) Two different types of receptors

b) More than two receptor types

c) Antagonism by different mechanisms

d) Hydrogen bonds or hydrostatic bonds

e) Covalent bonds

74. Beta carbolins is the example of:

a) A receptor block

b) An inverse agonist

c) Competitive antagonist

d) Irreversible antagonism

e) Inverse antagonist

75. which of the following is a partial agonist ?

a) Codeine

b) Diazepam

c) Morphine

d) Nicotine

e) Atropine

76.A partial agonist is best described as an agent that :

a) Has low potency but high efficacy


b) Acts as both an agonist and antagonist

c) Interacts with more than one receptor type

d) Cannot produce that full effect even at high doses

e) Acts only as an agonist at high doses

77. Cell membrane is a biological membrane composed of:


a) Protein molecules
b) Fatty acids
c) Ions
d) Polysacharides and proteins
e) Phospholipids and proteins

78. All drugs are absorbed by:


a) Passive diffusion
b) Facilitated diffusion
c) Active transport
d) Either facilitated diffusion or endocytosis
e) Either passive or active transport

79. Passive diffusion depends upon :


a) The charge on the particles
b) Large size of particles
c) Endothelial slits and hydrostatic pressure
d) Drug concentration and drug solubility
e) Hydrogen bonds
80. Active transport depends upon:
a) The concentration of the drugs
b) The hydrostatic pressure
c) Drug concentration and drug solubility
d) Water solubility, positive charge on the drug and pore size
e) A carrier and energy as ATP
81. Endocytosis is primarily:
a) Energy (ATP) dependant transport
b) A facilitated transport
c) Solubility dependant transport
d) A filtration process
e) A passive transport process
82. Partial agonist is:
a) The ligand which binds with half of its receptors
b) The drug that can only partially bind to its receptors
c) The drug that can bind at the inactive part of its receptors
d) The ligand that shows partial ionic and partial covalent bond to the receptors
e) The drug that shows a positve response but lesser than a full agonist response

83. An inverse agonist is a drug that:


a) Is inversely related to the dose
b) Shows a negative response as compared to that of an agonist
c) Brings the respons to a zero value
d) Decreases the number of receptors to zero level
e) That reverses the normal inside negativity and outside positivity
84.Regarding partial agonist :

a) It acts on same receptor system as the full agonist

b) It has a lower maximal efficacy regardless of dose

c) It may be less potent than full agonist

d) It may be pore potent than full agonist

e) All of them

85. Spare receptors are:


a) Receptors that increase the potency despite the presence of receptor antagonist.
b) Those receptors that are resistant to receptor antagonist
c) Receptors that give maximal response with less than 100% receptor occupancy
d) Receptors which render EC50 less than Kd
e) Receptors causing upregulation of receptors and increasing the effect
86. While using a drug with low therapeutic index one needs greater care because:
a) The drug has a greater degree of safety with therapeutic dose
b) The drug does not produce its therapeutic effect unless given in dose equal to or near to its toxic
level
c) The drug causes effects which are irreversible
d) The drug can only cause toxicity over a small range of doses
e) The drug can cause toxicity at high ED50
87. Plasma level monitoring of aspirin is not needed for its use because:
a) Aspirin has got a large therapeutic index
b) Aspirin causes a graded dose response relationship
c) Aspirin causes irreversible pharmacological effect
d) Aspirin is available without prescription
e) Aspirin is an orally administered drug
88. Which of the following statements drug efficacy is correct?
a) Efficacy decreases by the presence of a competitive antagonist
b) Drugs with high efficacy produce their effect at a lesser number of receptors binding
c) High efficacy is associated with high ED50 and low LD50
d) Receptor mediated signaling of maximal efficacy is at its maximal level
e) The efficacy is directly proportion to drug receptor binding dissociated constant.
89. Regarding the relationship of drug receptor interaction and potency of a drug which of following
statement is correct?
a) The higher the dissociation constant the greater is the potency
b) A competitive antagonist shifts the dose response curve to the right and decreases the potency
c) Drug with greater potency cause the shifting of curve to the right of a similar drug with lesser
potency
d) A high EC50 is associated with high potency
e) The higher the number of drug receptors complexes the greater is the potency
90. Which one of the following examples explains chemical antagonism.
a) Mannitol prevents the reabsorption of water by osmotically drawing it into the renal tubules
b) In hyperthyroidism a β-blocker drug dereases the tachycardia by binding to the β1 receptors
c) Fantanyl prevents the action of morphine by binding to μ opioid receptors
d) Digoxin anti bodies bind to digoxin and prevent it from inhibiting Na⁺- K⁺- ATP ase
e) Naloxone prevents the action of morphine by binding to μ opioid receptors
91. Regarding lesser quantity of lethal 3 metyhlfentanyl as copared to heroin which one of the following
statements is correct?
a) Kd of fentanyl is high
b) The affinity of 3 methyl fentanyl for μ opioid receptors is high
c) The LD50 of 3 methyl fentanyl is high
d) The EC50 of3 methyl fentanyl is high
e) The therapeutic index of fentanyl is high
92. Regarding lesser quantity of 25 mg ( one bag) lethal3 methyl fentanyl as copared to 75 mg of heroin
which one of the following statements is correct?
a) The LD50 of3 methyl fentanyl is 25 mg
b) The LD50 of 3 methyl fentanyl is 75 mg
c) The ED50 of3 methyl fentanyl is 25 mg
d) The therapeutic index of 3 methyl fentanyl is larger than that of heroin
e) 3 methyl Fantanyl is less potent than heroin
93. Treatment 3 methyl fentanyl toxicity naloxone administration causes withdrawal and then reversal of
effects, requiring multiple dose of naloxone because of its:
a) Inverse agonist action
b) Partial agonist action
c) Non competitive antagonist action
d) Reversible competitive antagonist action
e) Reversible agonist action
94.Which out of the morphine, 3-methyl fentanyl( potency 6000 times of morphine), pharmaceutical
fentanyl( potency 75 to 100 times of morphine) naloxone and naltrexone will have smallest EC50 and left
most drug dose response curve?
a) Naloxone
b) Naltrexone
c) Morphine
d) Fentanyl
e) 3-methyl fentanyl
95. A receptor effector coupling is:
a) Doubling the number of receptors for effector
b) Doubling the number of effectors for receptors
c) Doubling the magnitude of the response as compared with the number of receptors and
effectors
d) Transduction starting from drug binding to receptors and ending at the response
e) Both receptors and effectors become arranged in pairs of subunits
96. All of the following are capable of initiating a signal transduction process EXCEPT.

a) Combination of an agonist with its receptor

b) Combination of an antagonist with its receptor

c) Combination of a neurotransmitter with its receptor

d) Combination of a hormone with its receptor

e) None of them

97. When the magnitude of a drug response is greater than the magnitude of drug receptor interaction,the

phenomenon is called:

a) Second messenger activation


b) Receptor recruitment

c) Tachyphylaxis

d) Heterogenous desensitization

e) Signal amplification

98. The specific action of imatinib against the cells of chronic myloid leukemia is explained as follows;

a). Imatinib selectively acts on the tyrosine kinase associated receptors

b).Imatinib selectively inhibits BCR-AbI synthesis in hematopoietic precursor cells

c).Imatinib selectively binds the BCR-AbI protein of the tyrosine kinase receptors in abnormally growing

hemopoietic cells

d).Imatinib selectively degrades Philadelphia chromosomes in leukemic cells

e).Imatinib selectively acts on tyrosine kinase u

receptors

99. Interferon-α is also used for chronic myloid leukemia but as against imatinib it causes flu-like side

effects because:

a) Interferon-α stops the synthesis of BCR-AbI protein kinase

b) Interferon-α targets all myloid cells

c) Interferon-α acts on Philadelphia chromosome throughout the body

d) Interferon-α acts on BCR-AbI protein kinase in normally functioning cells

e) Interferon-α stops the synthesis of BCR-AbI production in all hematopoietic cells

100.Imatinib interrupts the activity of BCR-AbI protein because:

a) Imatinib inhibits the ability of BCR-AbI tophosphorylates substrates

b) Imatinib prevents the access of BCR-AbI to its target binding site by imatinib-plasma membrane

binding effect

c) Imatinib reverses the Philadelphia chromosomal mutation

d) Imatinib denature the protein and destroys the kinase receptor by binding to ATP-binding site of

BCR-AbI tyrosine kinase

e) Imatinib prevents the transcription of Philadelphia chromosome

101. BCR-AbI receptor tyrosine kinase affects intracellular signaling pathways by:

a) BCR-ABL receptor tyrosine kinase phosphorylates cytosolic proteins causing dysregulated cell growth

b) BCR-AbI receptor tyrosine kinase phosphorylates steroid hormones and cause upregulation of growth

transcriptional regulators of cell growth


c) BCR-AbI receptor tyrosine kinase removes phosphate groups from DNA and allows transcription

regulators access to specific genes

d) BCR-AbI receptor tyrosine kinase phosphorylates the tyrosine residue on cytoplasmic tail of the

receptor and opens cell membrane channels to allow the entery of growth regulators

e) BCR-AbI receptor tyrosine kinase removes phosphate groups from G- protein receptors thus

activating adenylyl cyclase signaling.

102. Which statement regarding G-protein – coupled receptors is correct?

a) These receptors are specific to acetylcholine

b) Ions flow occurs when transmembrane region opens

c) The extracellular domain consists of enzyme sites which upon binding of the ligand hydrolyze

guanosine triphosphate to diphosphate

d) The intra cellular region is linked to G-protein which upon binding of the ligand,affects signaling

molecules

e) There are five transmembrane subunits which upon binding of the ligand,release G-protein

103. The nicotinic acetylcholine receptor on neuromuscular junction is an example of:

a) Voltage gated extra cellular enzyme

b) Ligand gated tyrosine kinase receptor

c) Ligand gated G-protein coupled receptor

d) Ligand gated ion channel

e) Voltage gated ion channel

104. In organophosphorus poisoning the aged oxygen phosphorus bond is:

a) Hydrogen bond

b) Covalent bond

c) Ionic bond

d) Van der Waals interaction

e) Hydrophobic interaction

105. Organophosphorus compound action at the synapse is an action of:

a) Direct agonist

b) Indirect agonist

c) Inverse agonist

d) competitive reversible antagonist

e) Non competitive reversible antagonist

106. Organophosphorus compounds bind at:

a) Intracellular signal transduction enzyme


b) Transcription regulator

c) Cell surface adhesion receptor

d) Extracellular enzyme

e) Tyrosine kinase receptor

107. A drug which combines with at receptor and initiates a small degree of response, it also blocks receptors

and prevents the action of the other agonists is known as :

a. Agonist

b. Antagonist

c. Partial agonist

d. Both a and b

e. None

108. The difference between potency and efficacy is that :


a) Potency is the power of the drug while efficacy is the dose of a drug
b) Efficacy is the power o f a drug while potency is directly proportional to the dose
c) Potency is an indirect relation ship of dose to response and efficacy is a direct measurement
of response
d) Efficacy is usually taken on log scale while potency need not be taken on log scale
e) Potency is more important than efficacy
109.A drug R producing no response by itself causes the log dose response curve of another drug „S‟ to shift to

the right in a parallel manner without decreasing the maximal response. Drug „R‟ is a :

a) Partial agonist

b) Inverse agonist

c) Competitive antagonist

d) Noncompetitive antagonist

e) Placebo

110. The two drugs act on different type of receptors in a tissue and antagonize the action of each other,

which type to the following antagonism they present ?

a) Competitive antagonism

b) Pharmacological antagonism

c) Physiological antagonism

d) Chemical antagonism

e) Dispositional antagonism

111.A drug is said to be potent when:

a) It produces maximal response

b) The amount needed to produce a certain response is less

c) It produces minimum/no side effects

d) It is has a rapid onset of action

e) It has not distributed


112 . Drug A in a dose of 10mg produces same response as with 100mg of drug B:

a) Drug A is 10 times more potent than drug B

b) Drug B is 10 times more potent than drug A

c) Drug A is 10 times more efficacious than drug B

d) Both are equally potent

e) Both are equally efficacious

113.Graded dose response curve is :


a) A grading of decreasing doses and increasing responses
b) Increasing doses and decreasing responses
c) Increasing doses and increasing responses
d) Increasing doses for a constant ceiling response from the beginning
e) Increasing the grades of responses for a constant dose of different drugs of a group
114. Elimination half life is:

a) Directly proportional to the clearance

b) Indirectly proportional to the clearance

c) Indirectly proportional to the volume of distribution

d) The time taken by a drug in plasma to reduce to half of its inititial concentration

e) The half concentration of of the drug before its elimination

115. In a single compartment with elimination:

a) A steep curve with a plateau is obtained

b) A slow and steady rising curve is obtained

c) A slowly declining curve is obtained

d) A sharp rise and a slow decay curve is obtained

e) An exponential decline to new steady level is obtained

116. When drug attains an equilibrium between a vascular and an extravascular compartment:

a) There is a distribution phase followed by an elimination phase

b) There is an elimination phase followed by a distribution phase

c) The curve shows an exponential decline to a new steady level

d) There is a sharp rise and a slow decay curve

e) There is a steep rise and a slow steady decay curve

117. When there is drug distribution to an extravascular compartment and elimination:

a) A sharp rise and a slow decay curve is obtained

b) A continuous slow and gradual declining is obtained

c) A slow and steady rising curve is obtained

d) An exponential decline to a new steady level is obtained

e) A curve showing an early distribution phase and slow elimination phase is obtained

118. On stopping the drug the concentration in 3rd half life is:
a) 50% of original steady state concentration

b) 30% of original

c) 25% of original

d) 12.5% of original

e) 3.12%of original

119. After total hip replacement an 80 years old lady is put on both low molecular weight heparin and

warfarin. If she develops a urinary tract infection next year which of the following drug is responsible if she is

known to develop high fever, weakness, and skin rash within hours?

a) Acetaminophen

b) Sulfamethoxazole –trimethoprim

c) Rifampin

d) Levofloxacin

e) Vancomycin

120. After total hip replacement an 80 years old lady is put on both low molecular weight heparin and

warfarin. If she develops prosthetic joint infection with staphylococcus aureus only a few days later. She is

put on an aggressive 12 weeks course of combination of antibiotics in which I/V vancomycin and rifampin are

administered for two weeks followed by oral ciprofloxacin and rifampin for 10 weeks. What is the rationale of

this combination of therapy?

a) I/V vancomycin is less convenient which is later replaced by a more convenient oral medication of

ciprofloxacin

b) Amore potentially allergic combination has to be replaced with a less potentially allergic combination

c) O ne combination of antibiotics has to be replaced with another with greater synergistic effects

d) Vancomycin being more expensive, has to be replaced with a less expensive ciprofloxacin

e) To prevent the development of bacterial resistance the antibiotics have to be changed

121. After total hip replacement an 80 years old lady is put on both low molecular weight heparin and

warfarin. This combination is an example of:

a) An „oftarget‟ side effect of a therapeutic drug

b) A pharmacodynamics drug –drug interaction

c) A pharmacokinetic drug –drug interaction

d) An overdose of therapeutic drugs

e) Ahypersensitivity reaction to a therapeutic drug

122. After total hip replacement an 80 years old lady is put on both low molecular weight heparin and

warfarin. What is the rationale of coadministration of low molecular weight hweparin and warfarin in

immediate post operative period?

a) Achieving anticoagulant effect at lower doses and with minimal toxicity

b) Low molecular weight heparin at hospital has to be ultimately replaced with warfarin at home

c) Low molecular weight heparin is administered to achieve anticoagulation until plasma warfarin

concentration reaches a therapeutic anticoagulant level


d) Low molecular weight heparin is administered to compete with warfarin and slow its hepatic

metabolism to enhance therapeutic effect

e) Low molecular weight heparin and warfarin are molecular enantiomers which balance their

anticoagulant effects

123. Anepileptic lady on phenytoin becomes pregnant. She is so upset that she takes an overdose of

acetaminophen tablets. W hat is the effect of acetoaminophen on the fetus as it crosses the placenta?

a) The fetal hepatotoxicity by toxic metabolite

b) If the fetal liver is not developing at this stage,there will be no carcinogenic effects

c) If the fetal liver is developing at this stage , acetaminophen will cause teratogenic effects

d) Upon reaching the fetal circulation, acetaminophen will be conjugated to glutathione

e) If acetaminophen is conjugated by glucoronidation and sulfation, it will be trapped in the fetal

kidneys

124. The effect of chronic phenytoin use on acetaminophen metabolism in the liver is an example of:

a) An „off target‟ side effect

b) A pharmacodynamic drug interaction

c) A pharmackinetic drug interaction

d) A drug over dosage

e) A hypersensitivity reaction

125. What is the effect of chronic phenytoin use on acetaminophen intake?

a) Due to phenytoin over load liver will not be able to metabolize acetaminophen leading to liver

toxicity

b) Due to phenytoin over load liver will not be able to metabolize acetaminophen thus protecting from

liver toxicity

c) More efficiently conjugating toxic metabolite to glutathione thus protecting from liver toxicity

d) Due to phenytoin overdose liver enzymes induction will rapidly metabolize and clear

acetaminophen thus protecting from toxicity

e) Due to phenytoin overdose liver enzymes induction the more efficient metabolism of acetaminophen

will increase the chances of liver toxicity

126. Why there is a delay in time between the injestion of acetaminophen and onset of symptoms of

acetaminophen toxicity in the pregnant lady?

a) Delay in symptoms is due to large dose of acetaminophen taking a lot of time for absorption

b) Delay in symptoms is due to acetaminophen taking time to form covalent bonds with hepatic

metabolizing enzymes

c) Delay in symptoms is due to slowing of hepatic metabolism in pregnancy

d) Delay in symptoms is due to competing of acetaminophen with phenytoin for metabolism in the

liver

e) Delay in symptoms is due to the fact that clinical symptoms do not occur until glutathione stores

are depleted and toxic metabolite begins to bind and induce damage to hepatocytes
127. What is the mechanism of acetaminophen toxicity in the liver?

a) Development of cholestasis competitive blocking the metabolism of bile components by

metabolism of acetaminophen

b) Acetaminophen metabolites induce collagen deposition in the liver

c) Acting as an hapten ,acetaminophen induces autoimmune hepatitis

d) Acetaminophen is metabolized to a toxic metabolite that causes damage to the liver cells

e) Acetaminophen binds to a variety of other receptors in the liver

SYSTEMIC PHARMACOLOGY

DRUGS ACTING ON AUTONOMIC NERVOUS SYSTEM:


128. Contraction of vascular smooth muscle occurs by stimulating:
a) Beta2 receptors
b) Beta1 receptors
c) Alpha1 receptors
d) Muscarinic receptors (M2)
e) Dopamine receptors (D1)

129. Contraction of cardiac muscle can be inhibited by stimulating:


a) Beta2 receptors
b) Muscarinic M2 receptors
c) Alpha1 receptors
d) Beta1 receptors
e) Dopamine (D1) receptors
130. Selective beta1 agonist stimulation results in less tachycardia than non selective beta1 agonist stimulation
because:
a) Non selective agonist is less potent
b) Selective agonists are heart specific and do not block other receptors
c) Beta1 selective drugs inactivate alpha receptors
d) The vasodilation caused by beta2 receptors is avoided
e) Beta1 specific drugs cause upregulation of receptors
131. A patient using tricyclic antidepressants used adrenergic amine which resulted in unexpected rise of
blood pressure. What is the possible mechanism of this serious elevation of blood pressure?
a) Increased action of monoamino oxidase (MAO)
b) Decreased degradation of monoamine by metabolism
c) Increased tissue sensitivity to catecholamines.
d) Interference in uptake of amine by adrenergic nerve endings
e) Increased action of catechol-O-methyl transferase (COMT)
132. Which one of the following causes vasodilation by administration of an alpha adrenergic blocker?
a) Phenylephrine
b) Tyramine
c) Cocaine
d) Epinephrine
e) Dopamine
133. This drug causes increased peripheral resistance without causing cardiac stimulation:
a) Tyramine
b) Cocaine
c) Phenylephrine
d) Dopamine
e) Epinephrine

134. A βeta blocking drug is effective in treating essential hypertension because:


a) It blocks β receptors and stimulates vagal nerves
b) It causes reduction in both cardiac output and renin release
c) It inhibits entry of calcium into blood
d) It directly blocks angiotensin receptors
e) It produces EDRF.
135. βetareceptors agonists are used in bronchial asthma mainly due to their property of:
a) Inhibition of inflammatory response in the airways
b) Prevention of night awakenings
c) Relieving acute contraction of bronchial smooth muscles
d) Reduction of the effect of alpha agonists
e) Stimulation of bronchial secretion
136. A beta agonist with intrinsic sympathetic activity is sometimes preferred over a beta agonist without ISA
because: -
a) It prevents reflex tachycardia
b) It is less depressant of myocardium
c) Agonist without ISA undergo first pass effect
d) These have prolonged duration of action
e) These form a covalent bond with the receptors.

137. Alpha blocking drugs cause tachycardia most likely due to:
a) Inhibition of a simultaneous β activity
b) Inhibition of parasympathetic nerves
c) Release of epinephrine from adrenal medulla
d) Reflex stimulation of cholinergic nerves
e) Blockade of alpha2 receptors

138. When an inhaler containing a bronchodilator does not relieve asthma but increases the heart rate then
the appropriate line of action should be:
a) Immediately stopping the use of inhaler
b) A glucocorticoid in inhaler form is added
c) A glucocorticoid is given intravenously
d) Only a glucocorticoid inhaler is used
e) The inhaler drug is changed
139. A drug that should not be given to asthmatic patients?
a) Isoprenaline
b) Epinephrine
c) Atropine
d) Hyoscine
e) Propranolol

140. Which one of the following drugs is given to the patient with carbamate insecticide poisoning?
a) Atropine
b) Physostigmine
c) Neostigmine
d) Pilocarpine
e) Bethanecol

141. The short acting acetyl cholinesterase inhibitor is:


a) Ecothiophate
b) Pyridostigmine
c) Physostigmine
d) Neostigmine
e) Edrophonium
142. Which of the following sign corresponds to the agonist acetyl choline action?
a) Dry mucous membranes
b) Tachycardia
c) Bradycardia
d) Muscle fasciculations
e) Urinary retention
143. Which of the following can be a potential therapeutic use of physostigmine?
a) Nicotine poisoning
b) Pilocarpine poisoning
c) Pralidoxime poisoing
d) Muscarine poisoning
e) Scopolamine poisoning
144. Which one of the following is absolute contraindication of physostigmine administration?
a) Succinyl choline allergy
b) Bethanecol ingestion
c) Organophosphate poisoning
d) Tubocurare poisoning
e) Botulinum poisoning
145. Both myasthenia gravis and tubocurare poisoning produce similar symptoms because:
a) Both the conditions are associated with increased Ach degradation
b) Both the conditions are associated with weakness due to decreased number of available nicotinic
receptors
c) Both the conditions are associated with weakness due to antibody mediated destruction of Ach
d) Both the conditions are associated with weakness due to lack of presynaptic Ach release
e) Both the conditions are associated with weakness due to choline deficiency.
146. Autonomic ganglion blockade causes:
a) Vomiting and diarrhea
b) Urinary incontinence
c) Bradycardia
d) Vasodilation and hypotension
e) Vasoconstriction and hypotension
147. The effects of physostigmine in botulinum poisoned patient are mediated at which of the following
receptors?
a) NM
b) M4
c) M3 (M3mediate parasympathetic tone in glandular tissue and smooth muscle contributing to
sweating and diarrhea)
d) M2
e) M1
148. If a patient with botulinum toxicity is administered physostigmine which of the following would be the
clinical outcome?
a) Pupillary dilatation
b) Muscle fasciculations followed by paralysis
c) Sweating and diarrhea
d) Coma
e) Improved muscle strength ( botulinum poisoning would prevent any effect of physostigmine at the
neuromuscular junction)
149. How does botulinum toxin cause muscle weakness and prolonged paralysis?
a) It degrades synaptobravin and prevents the fusion of presynaptic storage vesicles with the cell
membrane
b) It reversibly binds to the SNARE proteins and blocks the fusion of presynaptic storage vesicles with
the cell membrane
c) It blocks Ca channels and Ca mediated exocytosis ofpresynaptic storage vesicles
d) It degrades Ach within presynaptic storage vesicles
e) It prevents the entry of Ach into presynaptic storage vesicles
150.A villager was exposed to insecticide spray and presents with frothing from mouth, bradycardia, pinpoint

pupils and convulsions. Which of the following drugs will you administer him?

a) Atropine

b) Atropine & physosigmine

c) Atropine & pralidoxime

d) Edrophonium

e) Pilocarpine

151.A patient is given non depolarizing neuromuscular blocker during open reduction of femur fracture. At

the end of surgery, the anesthetist restored neuromuscular transmission by administering:

a) Succinylcholine

b) Carbachol

c) Physostigmine

d) Neostigmine

e) Pralidoxime

152. A patient receives echothiophate during eye surgery. Which of the following enzymes is affected by this

autonomic drug?

a) Tyrosine hydroxylase stimulated

b) Acetylcholinesterase (ACHE) inhibited

c) Catechol-O-methyltransferase(COMT) inhibited

d) Monomine oxidase (MAO) stimulated

e) DOPA decarboxylase stimulated

153. A crop duster pilot has been accidently exposed to high concentration of organophosphate insecticide. If

untreated, the cause of death from such poisoning would probably be:

a) Cardiac arrhythmias

b) Gastrointestinal bleeding

c) Heart failure
d) Hypertension

e) Respiratory failure

154. Suxamethonium injected to a patient of which hereditary enzyme deficiency leads to prolonged skeletal
muscle relaxation?
a) Cytochrome oxidase
b) Glucose-6-phosphate dehydrogenase
c) Plasma cholinesterase
d) Liver transaminase
e) Hemo oxygenase
155. What is the mechanism of interaction of MAO inhibitors with red wine and aged cheese?
a) Red wine and aged cheese contain tyramine which acts directly on post synaptic adrenergic
receptors and cannot be metabolized in the presence of MAOIs.
b) Ethanol in red wine inhibits the reuptake of catecholamines which cannot be metabolized in the
presence of MAOIs
c) Red wine and aged cheese contain tyramine which which cannot be metabolized in the presence of
MAOIs and cause displacement of NE from the storage vesicles
d) Red wine and cheese directly block the inhibitory action of MAOIs
e) Aged cheese contains small amounts of amphetamine which cannot be metabolized in the presence of
MAOIs and cause displacement of NE from the storage vesicles
156. Hexamethonium is a sympathetic ganglion blocker. It blocks the:
a) Presynaptic α –adrenergic receptor
b) Skeletal muscle acetylcholine receptor
c) Postsynaptic ganglionic acetylcholine receptor
d) Presynaptic acetylcholine release
e) Post synaptic β- adrenergic receptor
157. Cheese reaction can preventedby the following physiologic action
a) Induction of the NE transporter
b) Inhibition of tyrosine hydroxylase
c) Induction of the aromatic L-amino acid transporter
d) Inhibition of the vesicular acetylcholine transporter
e) Inhibition of the vesicular monoamine transporter
158. A β- adrenergic receptor antagonist is not the proper drug for hypertension of cheese reaction because:
a) β- adrenergic receptor antagonist can block catecholamine action on the heart and lead to asystole.
b) β- adrenergic receptor antagonist can delay the normal metabolism of catecholamine
c) β- adrenergic receptor antagonist can block catecholamine action on the heart while it allows the
continued α-receptor stimulation at the peripheral blood vessels
d) β- adrenergic receptor antagonist can block catecholamine reuptake by NE transporter and cause
psychosis
e) β- adrenergic receptor antagonist can block catecholamine‟s CNS action and worsen the depression
159. Why Phentolamine is helpful in treating cheese reaction?
a) Being a non selective α- adrenergic agonist ,it competes with NE to reduce vasococonstriction and
blood pressure
b) Being a non selective α- adrenergic agonist ,it causes vasodilatation and sedation
c) Being a non selective α- adrenergic antagonist ,it promotes COMT- mediated metabolism of NE
and decreases blood pressure
d) Being a non selective α- adrenergic antagonist ,it prevents tyramine induced displacement of NE
and decreases anxiety
e) Being a non selective α- adrenergic antagonist ,it blocks the catecholamine induced
vasoconstriction and decreases blood pressure

160. Metoprolol differs from propranolol in which of the following respect?


a) Metoprolol is somewhat β2 selective
b) Metoprolol is less likely to cause airway constriction
c) Metoprolol is used for treatment of hypertension
d) Metoprolol inhibits renin release from the kidney
e) Metaprolol reduces the cardiac output to a much lesser extent
161. Phenylephrine is instilled into nasal mucosa for:
a) Blocking cholinergic receptors
b) Inducing vasoconstriction
c) Inducing vasodilation
d) Blocking histamine H1 receptors
e) Blocking beta receptors

162. The following drugs cause direct or indirect parasympathomimetic action except:
a) Pilocarpine
b) Atropine
c) Bethanecol
d) Neostigmine
e) Nicotine

163. Topical timolol instillation into the eye is meant for:


a) Increased out flow of aqueous humor
b) Miosis
c) Mydriasis
d) Decreased formation of aqueous humor
e) Cycloplegia

164. Which effect is likely to be reversed by neostigmine?


a) Paralysis of skeletal muscles by a depolarizing muscle relaxant
b) Pupillary miosis caused by bright light
c) Paralysis of skeletal muscles by a non depolarizing muscle relaxant
d) Bradycardia by vagal stimulation
e) Carbachol poisoning

165. Metaproterenol induced reflex change in heart rate can be blocked by administering:
a) Atropine
b) Neostigmine
c) Physostigmine
d) Prazosin
e) Propranolol

166. Which one of the following drugs on topical instillation into the eye would cause pupillary dilatation and
paralysis of ciliary muscles:
a) Atropine
b) Pilocarpine
c) Phenylephrine
d) Prazosin
e) Acetazolamide

167. Antimuscarinic drugs decrease the absorption of other drugs like morphine because these:
a) Chemically destroy morphine
b) Physically form a protective layer
c) Decrease the motility of G.I.T
d) Increase the motility of G.I.T
e) Decrease the Ph of G.I.T
168. Administration of which of the following drugs is most likely to cause bronchial muscle relaxation?
a) Clonidine
b) Norepinephrine
c) Phenylephrine
d) Timolol
e) Metaproterenol

169. Muscarinic receptor agonist drugs may cause vasodilation due to:
a) Histamine release
b) Norepinephrine
c) EDRF
d) Acetyl choline
e) Serotonin

170. Nicotine released at which of the following sites mimics acetylcholine action?
a) Adrenal medulla
b) S.A node
c) Circular muscles of iris
d) Smooth muscle of urinary bladder
e) Smooth muscle of GIT

171. Carbachol exhibits all of the following properties except:

a) It causes mydriasis
b) It contains carbonic acid ester bond
c) It is a parasympathomimetic drug
d) It has both nicotinic and muscarinic effects
e) It is resistant to acetylcholinesterase

172. All of the following are beta agonist drugs except:


a) Dobutamine
b) Phentolamine
c) Epinephrine
d) Isoproterenol
e) Norepinephrine

173. Which one of the following agents would restore a rapid heart rate to normal?
a) Phentolamine
b) Phenoxy benzamine
c) Propranolol
d) Isoproterenol
e) Edrophonium

174. Which of the following causes masking of the hypoglycemia of a diabetic patient on insulin?
a) Muscarinic agonist
b) Muscarinic antagonist
c) Alpha agonist
d) Beta agonist
e) Beta antagonist

175. A 50 year old asthmatic man comes in for a checkup and complains that he is having some difficulty in

starting to urinate. Physical examination reveals that the man has a blood pressure of 160/100 min Hg and a

slightly enlarged prostate. Which of the following medications would be useful in treating both of these

conditions?

a) Doxazosin

b) Labetalol

c) Phentolamine

d) Propranolol

e) Isoproterenol

176. The drug of choice to treat anaphylaxis is: -


a) Atropine
b) Diphenhydramine
c) Epinephrine
d) Norepinephrine
e) Isoproterenol
177. We administer a pharmacologic dose of epinephrine and observe (among other responses) a direct

increase of cardiac rate, contractility, and electrical impulse conduction rates. Which of the following

adrenergic receptors was responsible for these direct cardiac effects?

a) ᾳ 1

b) ᾳ 2

c) β 1

d) β2

e) β 3a

178. Sympathetic over activity can cause all of the following except:
a) Pupillary dilation
b) Tachycardia
c) Increased renal blood flow
d) Relaxation of bronchial muscles
e) Decreased intestinal molitility

179. Reading a news paper or a book requires adjustment of accommodation for near vision by ciliary muscle
contraction. This process is accomplished through the following second messenger:
a) cGMP
b) cAMP
c) DAG
d) Opening Na+ ion channel
e) IP3

180. Which one of the following is an indirect acting parasympathomimetic drug that readily crosses the blood
brain barrier?
a) Neostigmine
b) Physostigmine
c) Nicotine
d) Muscarine
e) Bethanecol

181. All of the following are indications for antimuscarinic drugs except:
a) Insecticide poisoning
b) Post operative spasm of bladder
c) Hypertensive crisis
d) Parkinson‟s disease
e) Motion sickness

182. All of the following instilled into the eye cause cycloplegia except:
a) Tropicamide
b) Scopolamine
c) Atropine
d) Cyclopentolate
e) Physostigmine
183. Atropine acts by antagonism of the following type:
a) Irreversible antagonism at muscarinic receptors
b) Chemical antagonism at muscarinic receptors
c) Reversible antagonism at muscarinic receptors
d) Physiological antagonism at nicotinic receptors
e) Competitive antagonism at nicotinic receptors
184. An infant treated for insecticide poisoning with atropine injections ultimately died of atropine overdose.
What could be the most likely cause of his death:
a) Heart block
b) Raised blood pressure
c) Hypotension
d) Dehydration
e) High grade fever

185. Salbutamol, a β2 agonist is expected to cause:


a) Dilation of skin blood vessels
b) Hypoglycemia
c) Renin release
d) Skeletal muscle tremors
e) Increased cAMP in mast cells

186. Phenylephrine, an alpha agonist drug causes:


a) Sweating
b) Constriction of vessels in nasal mucosa
c) Miosis
d) Increased HCl secretion in stomach
e) Increased skin temperature

187. Vasoconstrictor drug given with local anesthetics :


a) Prolongs the duration of anesthesia
b) Reduces the anesthetic effect due to vasoconstriction
c) Both blood spply and extent of local anesthesia reduce significantly
d) Increases both the blood flow and anesthetic effect
e) Has no effect on local anesthesia

188. Administration of noradrenaline causes reflex bradycardia. What will be the result of administration of
noradrenaline if the patient is first atropinized?
a) Bradycardia persists as before
b) Bradycardia worsens due to direct effect of the drug on heart
c) Bradycardia worsens but due to indirect reflex action on heart
d) The heart rate increases due to indirect reflex action
e) The rate increases due to direct action of the drug on heart
189. Adrenaline, dopamine and histamine combine with the respective receptors and produce their response

by one of the following mechanism.

a) Activating adenyl cyclase leading to increase intracellular cAMP level

b) Activating phospholipase

c) Inducing or inhibiting synthesis of ligand specific intracellular protein

d) Opening or closing ligand gated ion channels

e) Regulating intra cellular second messenger through G-protein couples receptors

190. The following drug readily causes mydriasis without the paralysis of ciliary muscles:
a) Tropicamide
b) Atropine
c) Norepinephrine
d) Phenylephrine
e) Pilocarpine

191. A girl aged 8 years complains of bed wetting problem. An indirect and long acting sympathomimetic
drug used orally provides relief from this problem. Which one of the following is possibly being used?
a) Phenylephrine
b) Ephedrine
c) Dopamine
d) Dobutamine
e) Epinephrine

192. Which one of the following drugs has the effects of increasing cardiac force of contraction, heart rate,
constriction of skin blood vessels and dilation of blood vessels of skeletal muscles?
a) Norepinephrine
b) Acetylcholine
c) Epinephrine
d) Isoproterenol
e) Metaproterenol

193. An adult male after receiving an injection of a drug, had increased systolic and diastolic blood pressures
but decreased heart rate and cardiac output which one of the following drug was most likely
administered?
a) Physostigmine
b) Bethanecol
c) Isoproterenol
d) Epinephrine
e) Phenylephrine

194. Dopamine is used as a pressor agent in cases of circulatory failure. All of the following are cardiovascular

effects of dopamine except :

a) Activation of β1 receptors in the heart

b) Increase the both systolic and diastolic blood pressure

c) Activation of dopamine receptors in the splanchnic area

d) Regulation of renal blood flow

e) Activation of ᾳ receptors at high doses

195. An old farmer with ischemic heart disease is also suffering from glaucoma. Which of the β-blocker agents
is correctly justified for his use.
a) Nadolol is free of β2 blocking effect
b) Esmolol is most beneficial for his heart
c) Metoprolol selectively blocks β2 receptors
d) Pinodolol has a local anesthetic effect as well
e) Timolol is free of the local anesthetic effect

196. Which of the following patients would not be benefitted by propanolol?


a. Hypertensive
b. Suffering from angina pectoris
c. One with heart block
d. One with stage fright
e. One with migraine
197. 56 year old man has hypertension and enlarged prostate; which biopsy shows to be benign prostatic hyperplasia? Which of the following drug would be the most

appropriate initial therapy?

a. Albuterol

b. Atenolol

c. Metoprolol

d. Prazosin

e. Timolol
198. The following effect of epinephrine can be blocked by phentolamine but not by propranolol.
a) Relaxation of the uterus
b) Tachycardia
c) Contraction of radial muscle of iris
d) Bronchodilation
e) Increase of cAMP in fat.
199. Which of the following drug combination could cause serotonin syndrome?
a) Metaproterenol and imipramine
b) Iproniazid and duloxetine
c) Phenyleophrine and amitriptyline
d) Phenylephrine and phenelzine
e) Cocaine and clonidine
200.What is the mechanism of action of clonidine as an antihypertensive?
a) Being a direct β₁- receptor antagonist ,it decreases the cardiac output
b) Being a direct α₂- receptor antagonist ,it decreases NE release and sympathetic tone
c) Being a direct α₂- receptor agonist ,it decreases the release of NE and sympathetic tone
d) Being a direct α₂- receptor agonist ,it decreases release of NE and sympathetic tone
e) Being a direct β₂- receptor agonist ,it causes vasodilatation
201. Which of the following will directly act to decrease the high blood pressure?
a) Turbutaline ,a β₂ agonist
b) Yohimbine ,an α₂ antagonist
c) Phentolamine , a non selective α antagonist
d) Methylphenidate, an amphetamine analogue
e) Phenylephrine ,an α₁ agonist
202. If a β-receptor antagonist is administered to cocaine induced hypertensive patient, what will be the
clinical outcome?
a) Hypothermia
b) Tachycardia
c) Increase in blood pressure
d) Bronchodilation in asthmatic
e) Hyperglycemia
203. What is the mechanism of cocaine poisoning?
a) Upregulation and increased sensitivity to both α and𝛽 receptors
b) Inhibition of MAO and accumulated catecholamines act on both α and𝛽 receptors
c) Cocaine is taken up into the synaptic vesicles of sympathetic neurons by vesicular monoamine
transporter and displaces NE which can then act on both α and𝛽 receptors
d) Cocaine inhibits NE transporter and prevents the reuptake of synaptic catecholamines which become
available to act on both α and𝜷 receptors
e) Cocaine is a direct agonist at both α and𝛽 receptors

DIURETICS:

204-1. Calcium stones may develop by using:


(a) Acetazolamide
(b) Mannitol
(c) Furosemide
(d) Chlorthiazide
(e) Spironolactone
204-2. Which of the following is drug of choice for management of neurogenic diabetes insidious?

a. Amiloride

b. Demedocyine

c. Desmopreasin

d. Hydrochlorothiazide

e. Lithium

204-3. A patient was prescribed one of the following drugs for ascities. After sometimes he complained or

development of Gynaecomastia, hyperkalemia, and obstruction to flow of urine. What drug of the following

could have those adverse effects?

a. Amiloride

b. Captopril

c. Triamterene

d. Spironolactone

e. Methyl dopa

205. Furosemide acts at:


(a) Proximal convoluted tubules
(b) Thick Descending limb of loop of Henle
(c) Thick Ascending limb of loop of Henle
(d) Distal Convoluted tubules
(e) Collecting ducts

206. Pulmonary edema is treated by high efficacy diuretics like:


(a) Acetazolamide
(b) Mannitol
(c) Furosemide
(d) Chlorthiazide
(e) Spironolactone

207. Hypokalemic periodic paralysis is treatable with:


(a) Acetazolamide
(b) Spironolactone
(c) Chlorthiazide
(d) Mannitol
(e) Furosemide
208. Hyperurecemia and worsening of Gout can be caused by:
(a) Furosemide
(b) Acetazolamide
(c) Mannitol
(d) Spironolqactone
(e) Chlorthiazide
209. Which of the following may worsen the benign prostatic hypertrophy?
a) Loop diuretics
b) Osmotic diuretics
c) Potassium sparing diuretics
d) ADH antagonists
e) Carbonic anhydrase inhibitors

210.A patient presents with severe pain and redness in left eye with rapid loss of vision for last 2 days. On

examination his intraocular pressure is very high. Which of the following intravenous drug is indicated for this

condition?

a) Acetazolamide

b) Frusemide

c) Mannitol

d) Pilocarpine

e) Timolol

211. Which of the following is likely to worsen a hearing disturbance?


a) Acetazolamide
b) Spironolactone
c) Mannitol
d) Hydrochlorothiazide
e) Furosemide

212. A 70 year old man is admitted with a history of recurrent heart failure and metabolic derangements.

He has marked peripheral edema and metabolic alkalosis. Which of the following drugs is most appropriate for

the treatment of his edema?

a) Acetazolamide

b) Digoxin

c) Dobutamine

d) Hydrochlorothiazide

e) Spironolactone

213. Concurrent use of most diuretics and NSAIDs causes inhibition of the diuresis except in the case of the
one of the following diuretic:
a) Mannitol
b) Furosemide
c) Chlorothiazide
d) Acetazolamide
e) Spironolactone
214. All except one of the following are indications for the use of thiazide diuretics. The exeption is:
a) Edema
b) Hypokalemia
c) Prevention of renal calcium calculi
d) Hypertension
e) Congestive cardiac failure
215. All of the following lead to increased secretion of renin except:
a) Decreased sodium in the renal cortex
b) Increased sodium at the distal tubules
c) Angiotensin II
d) Sympathetic over activity
e) Decreased blood pressure

216. All of the following drugs can lead to hypokalemia except:


a) Furosemide
b) Hydrochlorothiazide
c) Indapamide
d) Amiloride
e) Acetazolamide
217.Gynaecomastia, hyperkalemia, menstrual abnormalities are potential adverse effects of ?

a) Spironolactone

b) Amiloride

c) Captopril

d) Triamterene

e) Methyldopa

218. Which of the following condition is treated with acetazolamide?


a) Congestive cardiac failure
b) Hypertension
c) Cirrhosis liver
d) Edema
e) Glaucoma
219. Losartan acts by:
a) Inhibition of Na+ binding to the renal transporter
b) Inhibition of angiotensin II binding to its receptor
c) Inhibition of aldosterone binding to its receptor
d) Increasing the angiotensin II binding to calcium channels
e) Inhibition of renin release

220. Which one of the following drug can be used as an adjunct for chronic open angle glaucoma?
a) Spironolactone
b) Amiloride
c) Furosemide
d) Triamterene
e) Acetazolamide
221. A patient is put on normal doses of digoxin and some diuretic for his heart failure. He develops digoxin
toxicity. Which of the following may be responsible for the toxicity?
a) Hyponatremia
b) Hypochloremia
c) Hypokalemia
d) Hypocalcemia
e) Inhibition of digoxin metabolism
222.A patient who wants to go to high altitude for some project which drug he should take with himself for

light headedness

a) Paracetamol
b) Clarithromycin

c) Co-trimoxazole

d) Acetazolamide

e) Frusemide

223. A 52 year old man is suffering from congestive cardiac failure with pulmonary edema. Cardiologist has

advises a high efficacy diuretic which causes blockage of the Na/k/2Cl co-transport in thick ascending

limb.Which drug is prescribed ?

a) Thiazide

b) Triamterene

c) Mannitol

d) Urea

e) Furosemide

224. Which one of the following diuretics would you prefer to give in a patient with a high serum uric acid
level so that it increases the solubility of uric acid and prevents the formation of uric acid stones?
a) Hydrochlorothiazide
b) Acetazolamide
c) Spironolactone
d) Ethacrynic acid
e) Furosemide
225. A patient using a diuretic has developed high blood sugar and to some extent carbohydrate intolerance.
Which one of the following may be the cause?
a) Furosemide
b) Acetazolamide
c) Hydrochlorothiazide
d) Triamterene
e) Spironolactone
226. Which of the following diuretic should be avoided for concurrent use with gentamicin?
a) Triamterene
b) Spironolactone
c) Acetazolamide
d) Mannitol
e) Furosemide

227. Which of the following diuretic may further reduce the cardiac output in a patient of heart failure?
a) Mannitol
b) Ethacrynic acid
c) Chlorothiazide
d) Triamterene
e) Amiloride
228. A man received head injury in a road traffic accident and went into come. Which of the following
diuretics is indicated?
a) Furosemide
b) Acetazolamide
c) Amiloride
d) Mannitol
e) Triamterene
229. A man with renal insufficiency is investigated to have hyperkalemia. Which of the following diuretics is
the best choice for his use?
a) Spironolactone
b) Triamterene
c) Amiloride
d) Volsartan
e) Chlorothiazide
230. Ergometrine stops post-partum hemorrhage by:

a) Causing vasoconstriction of uterine arteries

b) Increasing the tone of uterine muscle

c) Promoting coagulation

d) Inducing platelet aggregation

e) All of the above

231.A patient presents with a history of frequent and severe headaches. When we give one of the commonly

used drugs for abortive therapy, sumatriptan, through which of the following agents is it mainly acting?

a) Histamine

b) PGF

c) Prostacyclin
d) Serotonin
e) Thromboxane

232. A group of college students is planning a mountain climbing trip to himalyas. Which of the following

drugs would be appropriate for them to take to prevent mountain sickness?

a) A thiazide diuretic

b) An anticholinergic

c) A carbonic anhydrase inhibitor

d) A loop diuretic

e) A β-blocker

DRUGS ACTING ON CVS:

233-1. Methyldopa causes lowering of blood pressure chiefly by


a) Decreasing cardiac contractility
b) Decreasing cardiac output40
c) Decreasing heart rate
d) Decreasing peripheral resistance
e) None of the above but by central neuronal inhibitors.
233-2. A patient was prescribed one of the following drugs for hypertension. He complamed of development

of dizziness, flushing, nausea, constipation and dependent edema. Which one of the following drugs could be

responsible for these adverse effects ?.


a. Nifedipine

b. Captopril

c. Verapamil

d. Clonidine

e. Nimodipine

234. Tricyclic antidepressant and clonidine used concomitantly:


a) Entrance the antihypertensive action of clonidine
b) Block the antihypertensive action of clonidine
c) Can cause aggrevation of depression
d) Not affect the action of cloniclonic
e) Cause orthostatic hypotension
235. What would be the most appropriate calcium channel blocker for a middle aged man suffering both from
hypertension and tachycardia?
a) Nifedipine
b) Amlodipine
c) Isradipine
d) Verapamil
e) Nicardipine

236. Which one of the following about cardiac glycosides is true?


a) These require a saturated lactone ring at position 17 for their biological activity
b) These are non competitive antagonists of steroids
c) These are rendered inactive when the sugar portion of molecule is removed
d) These are found in oleander, lily of the valley and numerous other plants
e) These are relatively non toxic unless taken with fats
237. A 65 years old man with chronic heart failure and a prolonged history of digoxin use has developed
digoxin toxicity. Concurrent use of which of the following may be the most likely cause of digoxin
intoxication?
a) Quinidine
b) Thiazide diuretics
c) Cholestyramine
d) Cimetidine
e) Enalapril
238. A patient with ventricular tachycardia gets transient slowing of sinus rate. For the sake of similar benefit
for the same type of tachycardia which drug of the following can be substituted?
a) Lidocaine
b) Atropine
c) Isoproterenol
d) Adenosine
e) Dopamine
239. A 65 years old patient of angina gets relief from transient bouts of a angina by using sublingual glycerol
trinitrate. Which factor is responsible for his relief with glycerol trinitrate?
a) β-adrenergic blockade
b) Phosphorylation of myosin light chains in smooth muscles
c) cGMP
d) Decreasing the ATP of the smooth muscles needed for contraction
e) Phosphodiesterase
240. Which of the following vasodilators causes sufficient venular smooth muscle relaxation in addition to
arteriolar smooth muscle relaxation to maintain cardiac output at appropriately lower level?
a) Diazoxide
b) Sodium nitroprusside
c) Hydralazine
d) Nifedipine
e) Minoxidil

241. Out of the following drugs which one should be contraindicated to use concurrently with the sildenafil in
a patient of erectile dysfunction as it may dangerously lower the BP to cause cerebral and cardiac
ischemia?
a) Nitroglycerine
b) Chlorothiazide
c) Captopril
d) Propranolol
e) Simvastatin
242. A man of 45 years is suffering from 2nd degree heart block in addition to his hypertension and transient
angina. Which one of the following drugs should specially be put on the warning list for him?
a) Prazosin
b) Enalapril
c) Losartan
d) Verapamil
e) Nifedipine
243. Which of the following adverse effects may be particularly associated with captopril?
a) Aplastic anemia
b) Hirsutism
c) Hepatitis
d) Hypokalemia
e) Proteinuria
244. Which one of the following drugs may cause severe constipation in a middle aged man suffering from
hypertension, chronic stable angina and hypercholestrolemia?
a) Labetalol
b) Atorvastatin
c) Lisinopril
d) Nitroglycerine
e) Verapamil

245. Which one of following may be responsible for dry cough with angiotensin converting enzyme inhibitors ?

a) Histamine

b) Serotonine

c) Prostaglandins

d) Substance p

e) Bradykinin

246. A young lady using niacin for her hyperlipoproteinemia, develops cutaneous flush. She is told by her
physician that it is simply a side effect of niacin which causes vasodilation. Which of the following
factors is causing this side effect?
a) Blockade of Ca channel
b) Release of histamine
c) Release of prostaglandins
d) Release of platelet derived growth factor (PDGF)
e) Release of EDRF
247. A 60 years old lady who is on digoxin for her chronic heart failure and atherosclerotic heart disease adds
another drug to her medications which results in bigeminal rhythm on ECG. Which of the following
drug has led to this problem?
a) Enalapril
b) Simvastatin
c) Nitroglycerine
d) Chlorothiazide
e) Phenobarbitone

248. The difference between linisopril and losartan is that the later drug:
a) Causes hyperkalemia
b) Is a drug of choice in pregnancy
c) Causes effective catecholamine blockade
d) Does not inhibit the synthesis of angiotensin II
e) Also inhibits the synthesis of angotensin I.
249. A middle aged house wife suffers from vasospastic angina. Which one of the following drugs is advisable
for relief and prevention of this type of angina?
a) Diltiazem
b) Aspirin
c) Simvastatin
d) Propranolol
e) Nitroglycerine
250.A 70 years old male is treated with S/L nitroglycerin for occasional bouts of effort induced angina. Which

of the following describes main mechanism?

a) Blocks alpha adrenergic receptors

b) Forms cyanide

c) Increases local synthesis and release of adenosine

d) Raise intracellular cGMP levels

e) Selectively dilates/ relaxes coronary arteries

251. A patient suffering from type I diabetes is also having high blood pressure. Which of the following drug
requires extra caution if used in this case?
a) Methyldopa
b) Propranolol
c) Captopril
d) Prazosin
e) Diazoxide
252. A patient is brought to the casualty with hypertensive crisis. He is promptly admitted for sodium
nitroprusside infusion. What other drug of the following should be administered to prevent the
production of a toxic metabolite:
a) Vitamin C
b) Vitamin K
c) Heparin
d) Sodium thiosulfate
e) Acetazolamide
253. The ultimate toxicity developing after prolonged use of nitroprusside is due to the following metabolite:
a) Nitric oxide
b) Cyanide
c) An alpha blocking agent
d) An irreversible renin inhibitor
e) A peroxide radical
254. A man aged 45 is using imipramine for his depression. He has been advised an antihypertensive because
his blood pressure remains persistently high. Which one of the following antihypertensive drug wound
have no desired effect on him due to drug interaction?
a) Hydralazine
b) Prazosin
c) Guanethidine
d) Atenolol
e) Minoxidil
255. Which of the following drug if given during pregnancy can cause renal damage in fetus?

a) Captopril

b) Diazoxide

c) Fenoldopam

d) Hydralazine

e) Thalidomide

256.You want to start ACE inhibitors therapy for a patient with essential hypertension. Which of the following

patient related condition contraindicates the use of an ACE inhibitor and so is ruled out before you present

this drug?

a) Asthma

b) Heart failure

c) Hyperlipidemia

d) Hypokalemia

e) Pregnancy

257. Drug of choice for treatment of S.V.T

a) Verapamil

b) Adenosine

c) Propranolol

d) Amiodarone

e) Digoxin

258. Which one of the following antihypertensives if used for a long period of time, may result in a positive
coombs test?
a) Doxazosin
b) Clonidine
c) Enalapril
d) Hyralazine
e) Methyldopa
259. Methyldopa acts by :
a) Inhibiting α₁receptors

b) Inhibiting β₁ receptors

c) Inhibiting α₂ receptors

d) stimulating α₁receptors

e) stimulating α₂ receptors

260. Propranolol :
a) blocks α₁receptors

b) blocks α₂ α₂ receptors

c) blocks β receptors

d) blocks 5HT receptors

e) blocks dopamine receptors

261. Verapamil is:


a) An α₁ blocker

b) An α₂ blocker

c) A Calcium channel blocker

d) A β₁ blocker

e) A β₂ blocker

262. Sodium nitroprusside is used in :


a) Myocardial infarction

b) Pheo chromocytoma

c) Cardiomyopathies

d) Cardiac arrhythmias

e) Hypertensive crisis

263. Captopril is:


a) Antagonist of β receptors

b) An ACE inhibitor

c) An agonist ofα receptors

d) An agonist of β receptors

e) Antagonist of α receptors

264. A 45 old man has recently been diagnosed with hypertension and started on monotherapy designed to

reduce peripheral resistance and prevent sodium and water retention. He has developed a persistent cough.

Which of the following drugs would have the same benefits but would not cause cough?

a) Losartan

b) Nifedipine

c) Prazosin

d) Propranolol

e) Methyl dopa
265. Which one of the following antiarrhythmic drugs is more selective for ischemic myocardium while it has a
minimal if any proarrhythmogenic effect on normal heart?
a) Lidocaine
b) Procainamide
c) Dysopyramide
d) Quinidine
e) Propranolol

266. Monitoring of PFTs, TFTs, renal function test needed for which antiarrhythmic drugs?

a) Digoxin

b) Amiodarone

c) Propranolol

d) Verapamil

e) Quinidine

267. Atenolol relieves chronic stable angina by:


a) Reducing the velocity of A-V nodal conduction
b) Decreasing the myocardial oxygen demand
c) Dilating the coronary arteries
d) Anti thrombotic effect
e) Reducing the peripheral vascular resistance

268. A young patient comes to the OPD with periodic paroxysmal supraventricular tachycardia? Which one of
the following drugs will be the most appropriate for prescription:
a) Adenosine
b) Nifedipine
c) Nitroglycerine
d) Verapamil
e) Lidocaine

269. Nitrate given for stable angina might sometimes cause paradoxical increase in myocardial oxygen

demand. The most effective drug in such circumstances will be:

a) Nifidipine

b) Metoprolol

c) Hydralazine

d) Hydrochlorthiazide

e) Prazocin

270. A lady aged 60 with graves disease develops arrhythmias secondary to the thyroid overactivity. Which
one of the following will help control this type of arrhythmia?
a) Digoxin
b) Bretylium Tosylate
c) Tocainide
d) Encainide
e) Propranolol
271. Na- Channel blockers include:
a) Propranolol
b) Amiodarone
c) Verapamil
d) Digoxin
e) Quinidine
272. The phase representing the initial rise in action potential is:
a) Phase zero
b) Phase one
c) Phase two
d) Phase three
e) Phase four
273. Sotalol is included in:
a) Na- Channel blockers
b) β- blockers
c) K- Channel blockers
d) Ca-Channel blockers
e) Miscellaneous group

274.Which of the following antiarrhythmic produces ganglion blocking effect ?

a) Quinidine

b) Procainamide

c) Disopyramide

d) Tocanxide

e) Phenytoin sodium

275. Arrhythmia is ;
a) No rhythm at all
b) Loss of heart function
c) Loss of myofibrils of heart
d) Loss of normal rhythm of heart
e) Loss of cardiac sounds

276. Cardiac action potential has the following number of phases in all:
a) Two
b) Three
c) Four
d) Five
e) Six
277. A Patient of acute myocardial infarction develops ventricular tachycardia. Which of the following is the
best drug in this case?
a) Digoxin
b) Atropine
c) Lidocaine
d) Dopamine
e) Quinidine
278. What is the appropriate choice of time in which the administered digoxin is supposed to reduce to half of
its plasma level in a healthy person:
a) 6 hours
b) 1.5 hours
c) 45 seconds
d) 1.5 days
e) 12 days
279. If a patient needs a drug to increase the force of contraction of the heart without significantly affecting
total peripheral resistance, the best drug would be:
a) Bethacholine
b) Epinephrine
c) Norepinephrine
d) Dobutamine
e) Salbutamol
280. Out of the following drugs affecting the heart function which one is most likely to produce arrhythmia as
its adverse effect?
a) Quinidine
b) Clonidine
c) Dobutamine
d) Lidocaine
e) Amlodipine
281. Which of the following drugs acts primarily by reducing ventricular preload?
a) Nifedipine
b) Hydralazine
c) Epinephrine
d) Nitroglycerine
e) Isoprenaline

282. Which of the following drugs activates the potassium current and by hyperpolarization alters the resting
potential of A.V node?
a) Verapamil
b) Adenosine
c) Xylocaine
d) Digitalis
e) Quinidine
283. A 66 years old man had a myocardial infarction. Which of the following drugs would be appropriate

prophylactic antiarrhythmic?

a) Lidocaine

b) Metoprolol

c) Procainamide

d) Quinidine

e) Verapamil

284. A patient having used procainamide for arrhythmia, develops toxicity due to ovredose. Which of the
following agent will help to treat the toxicity?
a) KCl
b) Ca-gluconate
c) EDTA
d) Sodium lactate
e) Digoxin
285. In treating digoxin over dosage, the life saving drug of choice is:
a) Phenytoin sodium
b) Magnesium
c) Fab fragments
d) Lidocaine
e) KCl

286. Which drugs of the following improves the symptoms but does not reduce mortality in chronic congestive
heart failure?
a) Carvedilol
b) Digoxin
c) Lisinopril
d) Enalapril
e) Spironolactone
287. Which one of the following agents given after digoxin facilitates digoxin toxicity:
a) Valsartan
b) Potassium
c) Magnesium
d) Captopril
e) Quinidine
288. A patient of chronic heart failure who has been receiving digoxin for a long time, suddently falls, victim
to organophosphorus poisoning. He is given atropine to save his life from the poisoning. Which of the
following action of the digoxin will be blocked?
a) Snow covered appearance of the objects
b) Gynaecomastia
c) Increased atrial contractility
d) Increased P – R interval
e) Tachycardia
289. The following drug is recommended specially for cerebrovascular accident due to hemorrhage:
a) Nifedipine
b) Nimodipine
c) Nitroglycerine
d) Verapamil
e) Hydralazine

290. A patient suffering from hypertension and angina, gets tachycardia whenever he uses the following drug.
Which one is the drug?
a) Propranolol
b) Isosorbide dinitrate
c) Diltiazem
d) Verapamil
e) Guanethidine

291. The following drug causes further renal damage in patients who are already suffering from renovascular
disease. It also causes renal damage in fetus if given in pregnancy?
a) Diazoxide
b) Methyldopa
c) Enalapril
d) Hydralazine
e) Verapamil

292.Which one of the following anti-hypertensive if used for a longer time results in a positive comb‟s test?

a) Clonidine

b) Doxazosin

c) Enalapril

d) Thiazine

e) Methyldopa

293.A non-selective beta blocker with low extraction ration, long half-life and ISA:

a) Atenolol

b) Propranolol

c) Metoprolol

d) Labetolol

e) Yohimbine

294. The following antihypertensive drug when given to the patient is most likely to cause orthostatic
hypotension:
a) Arteriolar dilator
b) α antagonist
c) β blocker
d) β1 agonist
e) ACE inhibitor

295. A 37 year old female with history of sulfonamide allergy and open angle glaucoma is treated with

topical timolol. She started having tightness in the chest afterwards, which was relieved with inhaled albuterol.

The likely explanation is:

a) Drug allergy to timolol

b) Hemolysis caused by G6PD deficiency

c) Bronchoconstriction due to β blockers

d) Induction of erythrocyte sickling

e) Timolol idiosyncrasy

DRUGS ACTING ON CNS:

296. Which one of the following drugs can be used for cough without any risk of drug dependence?
a) Dextromethorphan
b) Loperamide
c) Naltrexone
d) Codeine
e) Methadone
297. Diazepam is given in all of the conditions cited below except: -
a) Alcohol withdrawal
b) Panic disorder
c) Insomnia
d) Schizophrenia
e) Status epilepticus
298. Which of the following is helpful in alzheimer‟s disease?
a) MAO inhibitors
b) Dopamine receptor blockage
c) Antithrombotic drugs to dissolve cerebral arterial clots
d) Serotonin inhibitors
e) Acetylcholinesterase inhibitor

299. Which one of the following drugs may cause neuroleptic malignant syndrome?
a) Fluoxetine
b) Sodium valproate
c) Phenobarbitone
d) Chlorpromazine
e) Alprazolam
300. Which of the following drugs is likely to aggravate arrhythmia and may cause torsade depointes?
a) Mephenytoin
b) Na-valproate
c) Bromazepam
d) Thioridazine
e) Citalopram
301. Which of the following drugs can lead to myocardial infarction if given to a patient of ischemic heart
disease?
a) Fosphenytoin
b) Olanzapine
c) Pentazocine
d) Zolpidem
e) Olmitriptan

302. Which one of the following drugs is likely to cause gingival hyperplasia in a patient of epilepsy?
a) Valproic acid
b) Carbamazepine
c) Phenytoin
d) Gabapentin
e) Topiramate
303. Which of the following drugs is contraindicated in pregnancy because it can cause neural tube defects in
the infant?
a) Valproic acid
b) Primidone
c) Ethosuxamide
d) Phenobarbitone
e) Gabapentin
304. A patient comes to OPD with over dose toxicity of bromazepam. He has to attend the court. Which one
of the following can reverse his symptoms?
a) Naloxone
b) Amphetamine
c) Ephedrine
d) Flumazenil
e) Obidoxime
305. A 55 year old man who is a chain smoker has been advised to quit smoking by the doctor. Which of the

following drugs would you suggest will help him?

a) Venlafaxine

b) Duloxetine

c) Benzodiazepine

d) Bupropion

e) Nefazodone

306. Which one of the following can lead to orthostatic hypotension?


a) Clozapine
b) Olanzapine
c) Chlorpromazine
d) Fluphenazine
e) Molindone

307. Which of the following antipsychotic drugs can induce seizures in non epileptic patients?
a) Loxapine
b) Clozapine
c) Molindone
d) Droperidol
e) Fluphenazine
308. Which one of the following is the potential adverse effect of lithium?
a) Migraine
b) Orthostatic hypotension
c) Sexual dysfunction
d) Thyroid goiter
e) Psychosis
309. In patient with bipolar disorder fluoxetine can cause improvement in mood but may lead to a manic
episode because of:
a) Mere use of fluoxetine
b) Overdose of fluoxetine
c) Tolerance to fluoxetineC
d) Cure from depression
e) Hypersensitivity to fluoxetine
310.Why is their a delay of two or more weeks in the onset of therapeutic effect of SSRIs?
a) It takes several weeks for the storage and release of serotonin
b) It takes several weeks for autoreceptors to become desensitized to the increased synaptic serotonin
c) It takes several weeks for SSRIs concentrations to become therapeutic
d) It takes several weeks for SSRIs to increase serotonin synthesis
e) It takes several weeks for SSRIs to upregulate the post synaptic receptors
311. What is the mechanism of action of fluoxetine in relieving depression?
a) It increases the synthesis of serotonin from tryptophan
b) It increases the release of serotonin from presynaptic vesicles
c) It is an agonist at post synaptic serotonin receptors
d) It prevents the reuptake of serotonin
e) It prevents the metabolism of serotonion
312. Lithium toxicity can occur in patients having:
a) COPD
b) Hypothyroidism
c) Cerebrovascular disease
d) Renal insufficiency
e) Hepatic failure

313. A man is using antidepressant drugs. He is feeling uncomfortable with the drowsiness, dry mouth,
palpitation and feeling of fainting on upright posture. Which of the following drugs is most likely to
cause these problems?
a) Citalopram
b) Amitriptyline
c) Escitalopram
d) Venlafaxine
e) Fluoxetine
314.A hypertensive crisis is more likely to result from action of drugs from which one of the following drug

class ?

a) Tricyclic depresant

b) barbaturates

c) opoid

d) monoaminoxidase inhibitor

e) all of the above

315. A chemist and druggist who had lost her wife in an accident takes out the antidepressants from his
shelves and starts using the medicine as per direction over leaf. After a few days he develops headache,
nausea and palpitation after eating cheese and beens in his meal. He visits his doctor who finds that his
B.P has risen to 210/110mmHg. What medicine might have caused this?
a) Citalopram
b) Fluoxetine
c) Sertraline
d) Imipramine
e) Isocarboxazid
316. The increase in the potency of general anesthetics is associated with:
a)Increase in therapeutic index
b) Increase in its MAC
c) Increase in its oil solubility
d) Increase in its blood solubility
e) Increase in its solubility in cardiac muscle tissue

317. Which of the following general anesthetic increases the cardiac out put?
a) Ether
b) Thiopental
c) Ketamine
d) Propofol
e) Isoflurane
318.Which of the following drugs is both effective & safe to use in a pregnant patient suffering from bipolar

disorder?

a) Carbamazepine

b) Chlorpromazine

c) Lithium

d) Olanzapine

e) Valproic acid

319. Which of the following antiepileptic agents should not be given to a patient who is prone to psychosis
with the family history of schizophrenia?
a) Valproic acid
b) Phenytoin
c) Ethosuximide
d) Vigabatrin
e) Phenobarbitone

320. Which one of the following is accepted as the mechanism of action of vigabatrin?
a) It inhibits GABA aminotransferase
b) It blocks NMDA receptors
c) It blocks Na++ channels
d) It blocks Ca++ channels
e) It causes K+ channel opening

321. Which of the following is the drug of choice in trigeminal neuralgia?

a) Phenytoin

b) Methyldopa

c) Phenobarbitone

d) Carbamazepine

e) Levodopa

322. A young boy with absence seizures is being treated with ethosuximide. What is its mechanism of action?
a) Blocks T type calcium channels at thalamic neurons
b) It block Na+ channels at cortical level
c) It increases the chloride channel opening
d) It increases the GABA at synapses
e) It blocks NMDA receptors
323.Respiratory depression after the use of which of the following agents may be reversed by the

administration of flumezanil?

a) Desflurane

b) Fentanyl

c) Ketamine
d) Midazolam

e) Morphine

324. Which of the benzodiazepines is best suited for its rapid onset of action and short duration of action for a
patient undergoing oesophageal endoscopy?
a) Diazepam
b) Alprazolam
c) Oxazepam
d) Bromazepam
e) Midazolam
325. Which of the benzodiazepines is used only as a sedative and hypnotic agent ?

a) Clonazepam

b) Diazepam

c) Flurazepam

d) Alprazolam

e) Lorazepam

326. Which of the following anxiolytic has also antidepressant properties?

a) Buspirone

b) Chlordiazepoxide

c) Alprazolam

d) Clorazepate

e) Lorazepam

327. A man already using one of the following drugs develops headache, flushing, nausea and vomiting soon
after taking alcohol. Which of the following drug was already in his use?
a) Diazepam
b) Naloxone
c) Disulfiram
d) Moclobemide
e) Phenobarbitone

328. Benzodiazepines act by binding with:


a) Dopamine receptors
b) Serotonin receptors
c) C) PABA receptors
d) GABA receptors
e) NMDA receptors

329. The Benzodiazepine used to treat status epilepticus is:


a) Nitrazepam
b) Temazepam
c) Diazepam
d) Clonazepam
e) Triazolam

330.The Short acting Benzodiazepine is:


a) Triazolam
b) Chlordiazepoxide
c) Oxazepam
d) Clonazepam
e) Flurazepam

331. A minor and transient degree of analgesic effect is only found in:
a) Triazolam
b) Temazepam
c) Nitrazepam
d) Flurazepam
e) Diazepam

332. The Benzodiazepine used for the chronic treatment of seizures is:
a) Diazepam
b) Clonazepam
c) Flurazepam
d) Triazolam
e) Zolpidem

333. An alcohol addict after being convinced by a preacher abruptly takes the decision of withdrawal. He is
brought to the doctor with agitated behavior anxiety, visual hallucinations and disorientation in time
and space. Which one of the following should be prescribed initially to overcome the disturbing signs
and symptoms?
a) Chlorpromazine
b) Diazepam
c) Imipramine
d) Amphetamine
e) Disulfiram

334. A middle aged man after having been operated with complete blanced anesthesia, develops metabolic
acidosis, hyperthermia, muscle rigidity and hyperkalemia. Which one of the following is most likely to
have caused the problem?
a) Propofol
b) Fantanyl
c) Halothane
d) Ketamine
e) Pentothal
334-2. Treatment of malignant hyperthermia includes all of the following EXCEPT:

a. Cessation of anesthetic and changing of rubber tubing to anesthesia machine

b. Administration of dantrolene sodium 1mg/kg by rapid intravenous infusion until symptoms

subside

c. Hyperventilation of patient with 100% oxygen


d. Administration of fluids and diuretics to control yoglobmia and hyperkalemia

e. Lidocaine indicate for arrhythmias

335. Which of the following agents is associated with high incidence of disorientation, sensory and perceptual

illusions and vivid dreams on recovery from anesthesia?

a) Diazepam

b) Fentanyl

c) Ketamine

d) Midazolam

e) Thiopental

336. Which of the following anesthetic agents has best analgesic effect in low concentration?

a) Enflurane

b) Halothane

c) Isoflurane

d) Nitrous oxide

e) Ether

337. A patient is to undergo a brief general anesthesia for surgery. The safest and the fastest acting inhalation

general anesthetic would be :

a) Halothane

b) Isoflurane

c) Enflurane

d) Nitrous oxide

e) Nitrogen dioxide

338. A 50 years old patient with a history of heavy alcohol abuse present to clinic with complaints of dyspnea

on exertion and weakness. His lab studies reveal low Hb and increased spherocytosis. What agent should be

used to treat this anemia?

a) Folic acid

b) Iron

c) Vitamin B12

d) Vitamin B6

e) Erythropoietin

339.A 30 year old women presented with red and itchy eczematous dermatitis. She had a dental procedure

early in the day with a local anesthetic. She has a history of allergic reaction. Which of the following drug is

most likely involved?

a) Cocaine
b) Ropivacaine

c) lidocaine

d) Bupivacaine

e) Etidocaine

340.Which of the following inhalational anesthetics has a low blood gas partition coefficient but is not used for

induction of anesthesia because of airway irritation?

a) Desflurane

b) Influrane

c) Halothane

d) Isoflurane

e) Sevoflarane

341. Which of the following drugs may cause increase in intraocular pressure ?

a) Phenytocin sodium

b) Amoxicillin

c) Dopamine

d) Propranolol

e) Ketamine

342. Administration of local anesthetic causes disappearance and reappearance of different sensations in a
fixed sequence. What is the sensation first to disappear and last to reappear?
a) Temperature
b) Deep pressure
c) Autonomic efferent pathway function
d) Motor sensation
e) Pain sensation
343. Which of the following drugs is very effective blocker of sodium channels but tends the most active in

myocardial tissues with long action potentials?

a) Lidocaine

b) Quinidine

c) Amiodarone

d) Procainamide

e) Propranolol

344. Which of the following local anesthetics is useful for topical (surface) administration only?

a) Procaine

b) Bupivacaine

c) Benzocaine

d) Etidocaine

e) Lignocaine
345. Amide linked local anesthetics are primarily metabolized by:

a) Hepatic conjugation and glutathione

b) Tissue esterases

c) Renal dipeptidases

d) Hepatic microsomal cytochromeP450 enzymes

e) Pseudocholinestrases

346. Adrenaline is sometimes administered with lidocaine. What is pharmacological basis for this
combination?
a) Adrenaline makes the sodium channels more sensitive to the action of lidocaine
b) Adrenaline prevents degradation of lidocaine by inactivating tissue esterases
c) Adrenaline prevents the unbinding of lidocaine from sodium channels
d) Adrenaline- induced vasoconstriction maintains the concentration of lidocaine locally by slowing the
rate of removal
e) Adrenaline- induced vasoconstriction maintains a low pH locally to facilitate penetration of
lidocaine into the neurons
347. Lidocaine is an amide linked local anesthetic with rapid onset and medium duration of action. Its
pharmacokinetics are explained by its:
a) Moderate hydrophobicity allows its neuronal cell membrane penetration its remaining there and its
amide linkage prevents its degradation by esterases
b) High pKa allows large fraction of drug to be in a neutral form and able to enter the neuronal
membrane and its amide linkage prevents its degradation by estrases
c) Very high hydrophobicity allows it to easily cross the cell membrane and amide linkage allows its
tighter binding on sodium channels
d) Low pKa allows its larger fraction to be in neutral form and able to penetrate the neuronal
membrane. Its amide linkage maintains a low local pKa
e) Low hydrophobicity prevents it from being trapped within the neuronal cell membrane.Its amide
linkage prevents its degradation by esterases
348. Which of the following statements explains the pain relieving mechanism of lidocaine?
a) Lidocaine binds to the extracellular site on the sodium channel to block the sodium entery and
inhibits the activation of nociceptive,sensory,motor and autonomic fibers
b) Lidocaine binds to the extracellular pore of the sodium channel to block the sodium entery and
inhibit the activation of nociceptive A and C fibers
c) Lidocaine binds to the intracellular site on the voltage gated sodium channels, inhibits its activation
and blocks the propagation of action potentials in nociceptive A and C fibers
d) Lidocaine binds to the intracellular site on the voltage gated calcium channels, inhibits its opening
and blocks the propagation of action potentials in nociceptive C fibers
e) Lidocaine binds to the intracellular site on the voltage gated sodium channels, inhibits its activation
and blocks the propagation of action potentials in motor fibers
349.Rapid firing of impulses in the area of pain facilitates preferential blockage of nociceptive activation. This
phenomenon is called:
a) Tonic inhibition
b) Phasic inhibition
c) Hydrophobicity
d) Hypersensitivity
e) Epineural activation
350. About 30 minutes time is required for a topical local anesthetic to cross mainly :
a) Endoneurium
b) Perineurium
c) Epineurium
d) Stratum basale
e) Stratum corneum
351. Which of the following combination of statements is associated with local anesthetic toxicity?
a) Hypersensitivity: inhibition of Ca channels in plasma membrane
b) Convulsions: Excessive blockade of glutamate receptors
c) Heart block: sodium channel block in cardiac condution system
d) Hypersensitivity: amide- linked metabolism with allergic properties
e) Decreased contractility of heart:enhanced released of sodium from sarcoplasmic reticulum
352. In combined administration of spinal /epidural local anesthesia deveiopment of numbness with paralysis
of legs . Which one of the following statements is correct?
a) Injection of LA into the CSF inhibits the normal production of substance P, causing legs paralysis
b) Injection of LA into the epidural space preferentially inhibits motor impulses causing legs paralysis
c) Injection of LA into the CSF diffuses proximally and causes respiratory arrest
d) Injection of LA into the CSF inhibits normal glutamate transmission in the spinal cord,causing lower
extremity paralysis.
e) Injection of LA into the CSF penetrates spinal cord,inhibits motor impulses, causing leg paralysis
353. Which statement regarding lidocaine with adrenaline and tetracaine combined anesthesia is correct?
a) Tetracaine is more hydrophobic and is unable to diffuse away from the neuronal membrane and its
inhibition of calcium channels potentiates the inhibition of sodium channels provided by lidocaine
b) Tetracaine is more hydrophobic and it prolongs the duration of anesthesia and increases the potency
of this formulation
c) Lidocaine being more hydrophobic ,competes with tetracaine for penetration of the neuronal
membrane and decreases the time of onset of anesthesia
d) Lidocaine limits the local tissue ischemia by counteracting the vasoconstricting effect of adrenaline
e) Tetracaine is an ester linked anesthetic,so the amide linked lidocaine prevents its degradation by local
tissue esterases thus prolonging the duration of local anesthesia
354. The pharmacokinetic correlation of barbiturates to their effect is explained by which of the following
statements?
a) Barbiturates with an acidic pKa cannot be orally administered because of the risk of irritation of
gastric mucosa
b) The effect of barbiturates in the CNS is soon terminated by redistribution to other highly perfused
tissues
c) Barbiturates cause induction of microsomal enzymes which increase their metabolism and allow
them to act at lower doses
d) First pass effect of the orally administered barbiturates produce long acting and more potent active
metabolites
e) Highly lipophilic barbiturates rapidly cross BBB and exert a rapid and more sustained effect on CNS
355. One sign of barbiturate poisoning is:
a) Suppression of adrenal glucocorticoid release
b) Absence seizures
c) Loss of recent memory
d) Tachypnea
e) Pulmonary shunting
356. How does the old age affect the extent of CNS depression caused by barbiturates?
a) Elderly patients have increased absorption of barbiturates from GIT
b) Old age exhibits single compartment model of drug distribution in all sedative drugs
c) The renal clearance of phenobarbital is decreased in old age
d) The hepatic clearance of barbiturates is decreased in old age
e) The elderly patients are unable to accumulate lipophilic drugs due to their greater ratio of body fat
to muscle
357. The depression of CNS and respiration by concurrent effect of barbiturate and ethanol is due to:
a) Both the agents enhance the GABA affinity for GABAA receptors and inhibit the excitatory effects of
glycine
b) Both agents increase the synthesis and release of GABA
c) Barbiturate increase chloride ion influx and ethanol inhibits the glutamate excitatory effect on the
receptors
d) Both agents inhibit nor adrenaline synthesis in the CNS
e) Both the barbiturates and ethanol enhance the chloride influx and inhibit glutamate effect at its
receptors
358. what is M.O.A of barbiturates in inducing sleep and controlling epileptic fits?
a) Barbiturates increase the activation of NMDA receptor by glutamate
b) Barbiturates increase the duration of chloride channel opening at the GABAA receptors
c) Barbiturate decrease the activation of kainite receptor by glutamate
d) Barbiturates increase the duration of chloride channel opening at the GABAB receptors
e) Barbiturates increase the frequency of chloride channel opening at the GABAA receptors
359. Glutamate receptor antagonists may be effective in the treatment or prevention of
a) Muscle atrophy
b) Shizophrenia
c) Dementia
d) Sequelae of ischemic stroke
e) Spinal cord transection
360. Glutamate receptor activation is associated with:
a) Opening of chloride ion channels and increased in chloride efflux
b) Activation of phospholipase-C and increased influx of neurosteroids
c) Opening of Mg++ channels and increased Mg++ influx
d) Opening of Na+ and Ca++ channels increasing Na+ and Ca++ influx
e) Opening of K+ channels and increased K+ influx
361. Some GABA agonists are effective as sedatives while others are antiepileptics and others are more
effective general anesthetics?
a) GABA agonists with more specified clinical uses have their specifity in binding different GABA
subunits
b) GABA agonists with more specified clinical uses have their specific efficacy in opening chloride ion
channel
c) Orally administered agent are sedatives and antiepileptics while those administered I/V are general
anesthetics
d) The different effects are dose related
e) These are the agents which target only the GABAA receptors of the relevant effects produced by the
different parts of CNS

362. A boy of 10 years of age had to stay for night with his relatives. He was extremely embarrassed in the
morning due to nocturnal enuresis. He was taken to a doctor who advised methylphenidate. On the
very first day he took too much of the drug. He developed toxicity with one set of the following
findings. Guess what?
a) Bronchiolar constriction, hypotension
b) Convulsions, tachycardia, high blood pressure
c) Sedation, sluggish reflexes, diarrhea
d) Miosis, lacrimation, sweating
e) Loss of light reflex muscle, weakness, cold and clammy skin

363. A 30 years old women with negative pregnancy tests, starts discharging milk from her breasts and also
misses her periods. Which one of the following drugs may be responsible for her problem?
a) Iproniazid
b) Sertraline
c) Amoxapine
d) Paroxetine
e) Amitriptyline

364. You treat a 40 years old female for neurotic depression. After a few visits you prescribe amitriptyline to

relieve her depression. After 4 days she calls you to say the pills are having no effect and she still feels

depression. The best course of action would be to advise her to:

a) Continue on medication as prescribed as the drug often takes up 2-6 weeks to have its

antidepressant effect

b) Double the dose

c) Stop the pills and you will order other treatment from the pharmacist

d) Continue on the same dose and also start a second drug to sue in combination with it

e) Take the pills with an alcoholic drink to increase their effect

365.A patient is transported to emergency department after repeated episodes of fainting; the cause was

attributed to severe drug induced orthostatic hypotension due to alpha adrenergic blockade from one of the

drugs main side effects. Which of the following drugs was the most likely cause of this problem?

a) Buspirone

b) Chlorpromazine

c) Diphenylhydramine

d) Haloperiodol

e) Zolpiodem

366.Which of the following drugs is not used in mania ?

a) Heloperidol

b) Imipramine

c) Fluphenazime

d) Chlorpromazine

e) Carbamazepine

367. Out of the following drugs available, a schizophrenic patient wants to use a drug which relieves
particularly his negative symptoms of social withdrawal and emotional bluntness. Which one of the
antipsychotic drugs is the best choice?
a) Haloperidol
b) Chlorpromazine
c) Fluphenazine
d) Thiothixene
e) Olanzapine
368. A chemist and druggist comes across a patient with multiple bone fractures in an accident. The patient
has been given morphine injection by another quack. The chemist having the vast observation of large
scale sale of pentazocine for severe pains ,takes pity on the condition of the patient and injects
pentazocine. What will be the possible result?
a) Coma
b) Respiratory depression
c) Abrupt relief of pain
d) Worsening of pain
e) Convulsions
369.Bromocriptine is antagonist drug of:

a) Adrenaline

b) Prolactin

c) Somatotropin

d) Atropine

e) Amoxicillin

370. A man with some psychiatric problem, gets polydipsia and polyuria after receiving treatment for his
disease. Which one of the following drugs might have caused increased thirst and high input of urine?
a) Droperidol
b) Alprazolam
c) Lithium carbonate
d) Carbamezepine
e) Sertraline
371. A patient with abnormal and aggressive behavior and out burst of foul language is best treated with: -
a) Benhexal
b) Benztropine
c) Bromocriptine
d) Haloperidol
e) Fluoxetine
372. A 72 year old patient with parkinsonism presents with swollen feet, they are red, tender and very

painful .These symptoms would alleviate within a few days if the patient were to stop taking?

a) Amantadine

b) Benztropine

c) Bromocriptine

d) Levodopa

e) Selagiline

373. Which one of the following antipsychotic drugs is more likely to cause blurring of vision, dryness of
mouth, constipation and urinary retention?
a) Droperidol
b) Clozapine
c) Olanzapine
d) Risperidone
e) Chlorpromazine
374. Chlorpromazine can cause:
a) Tetanus

b) Vomiting

c) Hiccup

d) Parkinsonism

e) Schizophrenia

375. Chlorpromazine can be used for:


a) Schizophrenia

b) Mania

c) Depression

d) Parkinsonism

e) Anxiety

376.Taking levodopa in excessive amounts may cause:


a) Psychosis
b) Loss of balance
c) Inhibition of vomiting center
d) Intestinal ileus
e) Drowsiness and lethargy
377. Why protein diet should be avoided during levodopa therapy?
a) Protein prevents absorption of levodopa from G.I.T
b) Proteins compete with levodopa for transport across BBB
c) Protein enhance function of dopamine decarboxylase thus converting levodopa to dopamine
d) Protein prevent the transport of levodopa across BBB by binding to it in the blood
e) Protein prevent the transport of levodopa across BBB by binding to its transporter
378. What could be the clinical effect of parkinsonism?
a) Drug craving behavior
b) Emotional lability and bouts of laughing and crying
c) Inability to get out of a deep armchair
d) Inability to tolerate cold temperatures
e) Inability to move one side of the body
379. How do the drugs benztropine and trihexyphenidyl provide some beneficial effectin patients with
Parkinson,s disease?
a) These block the cholinergic inhibition of indirect pathway
b) These enhance the excitatory NMDA receptors,which stimulate the direct pathway
c) These modify the cholinergic striatal interneurons which regulate the direct and indirect pathways
d) These block excitatory NMDA receeptors in the striatum
e) These block cholinergic activation of dyskinetic muscle cells
380. Which of the following statements regarding the choice of therapy for parkinson‟s disease is correct?
a) In young patients dopamine agonists are used as effective initial treatment
b) Inhibitors of dopamine metabolism are used as initial therapy
c) Dopamine agonist therapy should be delayed till the development of tolerance to the effects of
levodopa
d) Because of its adverse effects ,levodopa should be started only when other options are not working
e) Because of its efficacy, levodopa is the drug of choice as soon as the diagnosis is confirmed
381. How will a patient of parkinsonism respond to levodopa change overtime?
a) Development of tolerance and requiring continuous infusions
b) Less frequent requirement of levodopa due to regeneration of dopaminergic neurons
c) Intermittent “on” and “off” periods requiring drug holidays from levodopa
d) Development of tolerance to levodopa and increased dose requirements
e) Development of sensitization to levodopa and and less frequent dose requirements
382. The effect of levodopa on parkinsonism is:
a) Development of alternative dopaminergic pathways
b) Stimulating dopamine synthesis in CNS
c) Dramatically improving symptoms
d) Restoring damaged dopaminergic neurons and reversing the disease
e) Preventing the progression but not reversing the disease
383. Selective loss of dopaminergic neurons result in symptoms of parkinsonism due to:
a) Loss of dopaminergic neurons in substantial nigra leads to decreased activity of indirect pathways
and movement inhibition
b) Loss of dopaminergic neurons in substantial nigra leads to decreased activity of direct pathways and
movement inhibition
c) Loss of dopaminergic neurons in striatum leads to release of GABA and movement inhibition
d) Loss of dopaminergic neurons in tuberoinfundibular pathways leads to release of GABA and
movement inhibition
e) Loss of dopaminergic neurons in circumventricular organs contributes to the inhibition of cortical
control of movement
384.Donepezil enhances alertness in dementia by enhancing the activity of:
a) Serotonin
b) Scopolamine
c) Caffeine
d) Acetylcholine
e) Dopamine
385. Glutamate exerts its effect by:
a) Preventing the uptake of catecholamine
b) Opening ligand gated ion channels
c) Inhibiting adenosine receptors
d) Promoting the release of encephalins
e) Preventing dopamine synthesis
386. Which one of the following drugs would reverse the muscle rigidity caused by strychnine?
a) An antihistamine
b) Phenytoin
c) A benzodiazepine
d) An MAO inhibitor
e) Levodopa
387. Peripheral adverse effects of levodopa, including nausea, hypotension, and cardiac arrhythmias, can be

diminished by including which of the following drugs in the therapy?

a. Amantadine

b. Bromocriptine

c. Carbidopa

d. Entacapone
e. Ropinirole

388. Which of the following is the adverse effect of chlorpromazine?


a) Insomnia

b) Tardive dyskinesia

c) Alzheimer,s disease

d) Pruritus

e) Senile dementia

389. Newer atypical drugs cause:


a) More extrapyramidal symptoms

b) Lesser extra pyramidal effects

c) More schizophrenic symptoms

d) Mania

e) More hypotension

390. Antipsychotic drugs are:


a) D₁ receptor,s agonist

b) D₁ receptor antagonists

c) α₁ receptor,s agonists

d) β₁ receptors agonists

e) D₂ receptors antagonists

391. Which one of the following antipsychotic drugs is most likely to cause drug induced parkinsonism?
a) Ziprasidone
b) Haloperidol
c) Olanzapine
d) Clozapine
e) Sertindole
392. Which of the following is most likely to cause neuroleptic malignant syndrome due to antipsychotic
therapy
a) Chlorpromazine
b) Quetiapine
c) Amantadine
d) Haloperidol
e) Clozapine
393. Which of the following is an example of extrapyramidal effect?
a) Neuroleptic malignant syndrome
b) Galactorrhea
c) Sedation
d) Orthostatic hypotension
e) Shuffling gait
394. What is the potential risk to a patient who concurrently takes amphetamine and tranylcypromine?
a) Seizures
b) Liver failure
c) Loss of peripheral vascular tone
d) Muscle rigidity and symptoms of parkinsonism
e) Bradycardia
395. What is rationale behind prochlorperazine‟s antiemetic action?
a) Stimulating D₂ receptors in nigrostriatal system
b) Blocking muscarinic receptors in GIT
c) Blocking D₂ receptors in area postrema
d) Blocking D₂ receptors in mesolimbic system
e) Blocking D₃ receptors in mesocortical system

396. A young man suffering from schizophrenia, develops high fever, muscle rigidity and tremors with
antipsychotics. His serum creatinine kinase CK is also elevated. What has happened to him:
a) Drug induced parkinsonism
b) Tardive dyskinesia
c) Drug allergy
d) Over dose toxicity
e) Neuroleptic malignant syndrome
397. Which of the following can be caused by meperidine only and not morphine?
a) Respiratory depression
b) Increased pain threshold
c) Pin point pupils
d) Psychosis and seizures
e) Constipation

398. Morphine is not advisable in a patient with:


a) Hypertension
b) Ischemic heart disease
c) Lung edema
d) Heroin addiction
e) Head injury

399. Which one of the following drugs undergoes elimination by zero order kinetics?
a) Phenytoin sodium
b) Thiopentone sodium
c) Pentazocine
d) Oxazepam
e) Prochlorperazine
400.A 25 years old female with absence seizures is treated with ethosuximide. Which of the following is the

principal mechanism of action of ethosuximide?

a) Calcium channel blockade

b) Increase in frequency of the chloride channel opening

c) Increase in GABA

d) Increased potassium channel permeability

e) Sodium channel blockade

401. Selective serotonin receptor inhibitors have an advantage over the tricyclic antidepressants because these
are relatively free of:
a) Convulsions
b) Nephrotoxicity
c) Anticholinergic effects
d) Hepatotoxicity
e) Sexual disturbances

402. The first I/V drug preferably given to a patient of status epilepticus is:
a) Phenytoin sodium
b) Diazepam
c) Carbamazepine
d) Na-valproate
e) Phenobarbitone
403.After a few weeks on a drug, a patient reports poly dypsia will polyuria.Which of the following drugs

most likely responsible for the sign and symptoms?

a) Fluoxamide

b) Diazepam

c) Haloperiodol

d) Lithium

e) Phenytoin

404. The justification of diphenhydramine given as adjunct to anti parkinsonism drugs is:
a) To prevent hypomania
b) To counteract sedation
c) To overcome dystonic reactions
d) To counter the skin allergy of antiparkinsonism drugs
e) To increase the antagonism of acetylcholine

405. Which of the following local anesthetic drugs does not contain a hydrophilic amine group and is therefore
used only for topical application on the mucous membranes?
a) Rupivacaine
b) Benzocaine
c) Procaine
d) Lidocaine
e) Bupivacaine

406. The only antiepileptic drug which can cause toxicity of other drugs due to inhibition of their metabolizing
enzyme is:
a) Ethosuximide
b) Phenytoin sodium
c) Phenobarbitone
d) Carbamazepine
e) Sodium valproate
OPIOID ANALGESICS:
407. Regarding the use of tramadol which one is true?

a) Has beta blocking properties

b) Blocks serotonin reuptake

c) Has greater opioid activity than morphine


d) Is directly inhibited by yohimbine

e) It raises the seizure threshold

408.In case of opioid overdose, naloxone can be given in repeated doses because of which property of

naloxone?

a) May have a shorter half -life than the opioid agonist

b) Is only effective in high cumulative doses

c) Is needed to stimulate the respiratory Centre

d) Is safe only in extremely small doses

e) Is only a partial agonist

409. The drug not used for analgesia in a patient of head injury.

a) Morphine

b) NASIDS

c) Rofecoxib

d) Acetaminophen

e) Aspirin

410. Opioid receptors are G-protein coupled receptors that act by:
(A) Opening up Na channels and closing down Ca channels
(B) Closing the Na channels and opening up of K channels
(C) Opening up Ca channels and closing down o f Na channels
(D) Opening up of Ca channels or closing down of K channels
(E) Opening up of K channels and closing down of Ca channels

411. Opioids do not induce µ receptor endocytosis exept:


a) Morpine
b) Codeine
c) Methadone
d) Buprinorphine
e) Nalbuphine

412. Tolerance to all effects of Morphine occurs:


a) After more than 6-8 months of use
b) Quite rapidly
c) After several weeks of use
d) Except to miosis and constipation
e) Except euphoria and analgesia

413. Morphine is used in dyspnea of pulmonary edema and heart failure because:
a) Morphine slows respiration and reduces air hunger
b) Morphine relieves anxiety and causes vasodilation
c) Morphine relieves the pain of dyspnea
d) Morphine relaxes the muscles of respiration
e) Morphine relieves the cardiac function in heart failure
414. For morphine withdrawal in addicts the drug used is:
a) Naloxone
b) Naltrexone
c) Nalbufine
d) Nallorphine
e) Methadone
NSAIDs:

415. All of the following are reversible COX inhibitors except:


a) Ibuprofen
b) Mefenamic acid
c) Aspirin
d) Diclofenac K
e) Piroxicam

416. Which one of the following is a leucotriene inhibitor?


a) Celecoxib
b) Zafirleukast
c) Nimesulide
d) Meloxicam
e) Aspirin
417.Patients with normal platelet count and normal bleeding time may still bleed as a result aspirin ingestion

prior to the dental or surgical procedures. The aspirin interference with the normal platelet function may last

as long as:

a) 4 hours

b) 12 hours

c) 2 days

d) 5 days

e) 7 days

418. Which one of the following has a potential for increasing the risk of myocardial infarction?

a) Aspirin
b) Phenylbutazone
c) Indomethacin
d) Zafirleukost
e) Celecoxib

419. Which one of the following competes aspirin for tubular secretion?
a) Alcohol
b) Probenecid
c) NH₄Cl
d) NaHCO₃
e) NaOH

420. All of the following cause gastric irritation except:


a) Aspirin
b) Ibuprofen
c) Indomethacin
d) Piroxicam
e) Nimesulide

421. More lower G.I irritation occurs :


a) With Cox- 2 inhibitors
b) With non acetylated drugs
c) With Diclofenac
d) With Indomethacin
e) With drugs undergoing entero hepatic circulation

422. NSAIDs cause:


a) Increased leukocyte migration
b) Increased lymphokine production
c) Decreased blood vessel sensitivity to histamine
d) Production of free radicals
e) Production of bradykinin

423.A patient was transported by ambulance to emergency department took a potentially lethal overdose of

aspirin. Which of the following drugs would be helpful to manage this severe aspirin poisoning?

a) Acetaminophen

b) NaHCO3

c) Amphetamine

d) N-acetyl cysteine

e) Phenobarbital

424. In patients of arthritis with asthma and bleeding tendencies:


a) Only aspirin is advisable
b) Ibuprofen is a safe choice
c) Acetylated NSAIDs are preferred
d) Non acetylated NSAIDs are used
e) COX-1 inhibitors should be given

425. Which one of the following drug is given for patent ductus arteriosus in infants?
a) Indomethacin
b) Aspirin
c) Sulindac
d) Piroxicam
e) Celecoxib

426. Which of the following is the best for familial intestinal polyposis
a) Diclofenac
b) Sulindac
c) Aspirin
d) Ibuprofen
e) Piroxicam
427. Which one of the following drug acts in gout by binding to intracellular protein tubulin, preventing its
polymerization, leukocyte migration and phagocytosis?
a) Sulficphyrazone
b) Aspirin
c) Colchicine
d) Indomethacin
e) Allopurinol

428. Gold compounds modify the rheumatoid arthritis process and:


a) The injections are non toxic and therefore tolerated well
b) These are only administered parenterally
c) The drugs clinical response occurs in 24 to 48 hours
d) It acts by inhibiting the function of macrophages
e) If the response does not occur in one week the drug must be discontinued
428-1. Which one of the following drugs can be used as disease modifying anti rheumatic drug.

a. Aspirin

b. Lefunomide

c. Colchicine

d. Ibuprofen

e. Niclosamide

429. Which one of the following statements regarding NSAIDs is true?


a) The best choice of NSAIDs is made on the basis of analgesic versus antipyretic action
b) Preferably an NASAIDs without gastric irritating effect is prescribed
c) All NSAIDs irreversibly inhibit platelet aggregation
d) All NSAIDs except aspirin are reversible inhibitors of platelets
e) Patient with some renal problem should be advised only non nephrotoxic NSAIDs
430.Which one of the following drug is given for the closure of patent ductus arteriosus in infants?

a) Aspirin

b) Celecoxib

c) Indomethacin

d) Piroxicam

e) Sulindac

431A 45 years old man presents to the emergency ward with aspirin overdose. Which of the following will

result from aspirin overdose?

a) Causes metabolic & respiratory acidosis

b) Causes metabolic & respiratory alkalosis

c) Causes metabolic alkalosis & respiratory acidosis

d) Causes metabolic acidosis

e) Causes respiratory acidosis

432. Regarding ibuprofen:


a) Its action is potentiated by addition of aspirin
b) It causes more gastrointestinal irritation as compared to aspirin
c) The dose given as antipyretic does not have full anti-inflammatory action
d) It is excreted unchanged in the urine
e) It is less potent than aspirin if used as an anti-inflammatory agent
433. The best treatment of headache with aspirin is achieved when: -
a) Aspirin is enteric coated
b) Plain aspirin (regular)
c) Buffered aspirin
d) Aspirin + mesoprostol
e) Aspirin + Femotidine
434. Aspirin toxicity may range from common therapeutic mild and moderate to severe and lethal degrees.
Metabolic acidosis resulting from aspirin toxicity is graded as:
a) Mild
b) Common therapeutic side effect
c) Moderate
d) Severe
e) Lethal
435. Reye‟s syndrome is caused by :

a) Aspirin

b) Paracetamol

c) Allopurinol

d) Probenecid

e) Colchicine

436.Regarding aspirin which of the following statement is true?


a) It is an effective remedy for visceral pain
b) It is an analgesic drug but does not have an anti-inflammatory effect
c) Very high doses of aspirin may exacerbate the gouty arthritis
d) Aspirin inhibits the synthesis of prostacycline recurring its protective effect
e) Antiplatelet effect of aspirin is reversible
437. Which statement about the cardiac prophylactic use of aspirin is correct for a patient of hemophilia and
peptic ulcer?
a) He should use a buffered form of aspirin
b) He should take only moderate dose of aspirin
c) He should use acetoaminophen instead of aspirin
d) He should use CoX-2 inhibitor like celecoxib
e) He should abstain from using any drug owing to risk of more harm than benefit

438. For a child with 102oF fever of viral respiratory disease which of the following should be used:
a) Small pediatric dose of aspirin
b) Cold sponing water bath
c) A CoX-2 inhibitor
d) Acetaminophen for antipyretic effect
e) Nothing but wait and see

439. More lower G.I irritation occurs :


a) With Cox- 2 inhibitors
b) With non acetylated drugs
c) With Diclofenac
d) With Indomethacin
e) With drugs undergoing entero hepatic circulation

440. NSAIDs cause:


a) Increased leukocyte migration
b) Increased lymphokine production
c) Decreased blood vessel sensitivity to histamine
d) Production of free radicals
e) Production of bradykinin
441. In patients of arthritis with asthma and bleeding tendencies:
a) Only aspirin is advisable
b) Ibuprofen is a safe choice
c) Acetylated NSAIDs are preferred
d) Non acetylated NSAIDs are used
e) COX-1 inhibitors should be given

442. Which one of the following drug is given for patent ductus arteriosus in infants?
a) Indomethacin
b) Aspirin
c) Sulindac
d) Piroxicam
e) Celecoxib

443.Which of the following is the best for familial intestinal polyposis?


a) Diclofenac
b) Sulindac
c) Aspirin
d) Ibuprofen
e) Piroxicam
444. All of the following are reversible COX inhibitors except:
a) Ibuprofen
b) Mefenamic acid
c) Aspirin
d) Diclofenac K
e) Piroxicam

445. Which one of the following is a leucotriene inhibitor?


a) Celecoxib
b) Zafirleukost
c) Nimesulide
d) Meloxicam
e) Aspirin
446. Which one of the following has a potential for increasing the risk of myocardial infarction?
a) Aspirin
b) Phenylbutazone
c) Indomethacin
d) Zafirleukost
e) Celecoxib
447. Which one of the following competes aspirin for tubular secretion?
a) Alcohol
b) Probenecid
c) NH₄Cl
d) NaHCO₃
e) NaOH
448. All of the following cause gastric irritation except:
a) Aspirin
b) Ibuprofen
c) Indomethacin
d) Piroxicam
e) Nimesulide

449. In acute gout ,one or two days colchicine may be the correct choice for the pain except in patient:
a) Having fatty change in liver
b) Bronchospasm
c) Myelosuppression
d) Vomiting and diarrhea
e) Tachycardia
450. After several weeks of treatment, hyperuricemia is significantly reduced. Which of the following drugs
might have resulted in reduced level of uric acid:
a) Probenecid
b) Colchicine
c) Ibuprofen
d) Acetaminophen
e) Allopurinol

451. If pobenecid is administered to a patient with a mild degree of hyperuricemia but not apparent
symptoms the patient may develops symptoms of gouty of arthritis in a few days. What is the primary cause
of it?
a) Reduced excretion of uric acid in urine
b) Increased uric acid synthesis
c) Idiosynacracy
d) Uric acid forms crystals with probenecid
e) Probenecid increases crystal formation by causing systemic acidosis

452. Which of the following drugs is used for the treatment of acute gouty arthritics?

a) Colchicine

b) Probenicid

c) Paracetamol

d) Cold

e) ibubrufen

453. Aspirin aggravates the gout if taken concurrently with which of the following drugs:
a) Colchicine
b) Probenecid
c) Alcopurinol
d) Indomethacine
e) Piroxicam
454. Neonates having a patent ductus arteriosus can be treated with which agent to induce a relatively rapid

closure and thus often avoid surgical intervention?

a) Phenobarbital

b) Indomethacin

c) Hydrochlorothiazide

d) Prostaglandin E1

e) Epinephrine

455. Uric acid causes gouty arthritis by:


a) Directly activating leukotriene by receptors
b) Uncoupling of oxidative phosphorylaction and causing tissue hypoxic necrosis
c) Stimulating formation of microtubules in leukocytes
d) Causing enhanced activity of tumor necrosis factor TNF
e) Mechanically damaging articulating surfaces of joints

456. Which one of the following enzyme is essential for uric acid synthesis?
a) Cyclooxygenase 1
b) Cyclooxygenase 2
c) Xanthine oxidase
d) 5-lipoxygenase
e) Phospholipase – C
457. Which of the following is the adverse effect of cholchicine?
a) Diarrhea
b) Vomiting
c) High uric acid concentration in urine
d) Hypersensitivity reactions
e) Seizures

458. The COX-2 inhibitor has an advantage over an non selective COX inhibitor is the fact that: -
a) Its action starts rapidly
b) It treats the disease rather than only relieving the pain
c) It has a lower risk of G.I.T irritation
d) It decreases the synthesis of uric acid
e) It is less nephrotoxic than non selective COX inhibitor
459. Penicillamine differs from aspirin in the treatment of rheumatoid arthritis that:
a) It stops, slows or probably reverses the arthritic process
b) It is free of toxic effects
c) It quickly relieves the pain
d) It stimulates the inflammatory responses
e) It is first line therapy for gout

460. The antidote for initial management of acetaminophen toxicity is:


a) Cyanocobalamine
b) N-acetylcysteine
c) Acetylcholine
d) Warfarin sodium
e) Atropine
461. The cause of death in acetaminophen poisoning is:
a) Heart block
b) Status asthmaticus
c) Metabolic alkalosis
d) Hepatic failure
e) Renal failure
462. Which one of the following is useful in the management of aspirin poisoning?
a) Sodium bicarbonate
b) Ammonium chloride
c) N-acetyl cysteine
d) Phenobarbitone sodium
e) Dextroamphetamine

463. A patient has taken high doses of aspirin. Which one of the following clinical features will show the rising
blood levels of aspirin?
a) Myopia
b) Pin point pupil
c) Tinnitus
d) Cynosis
e) Hypertension
464. Aspirin causes its antiplatelet effect by:
a) Blocking platelets receptor for ADP
b) Inhibiting thromboxane A2 synthesis
c) Increasing thrombolysis
d) Stimulating antithrombin III
e) Inhibiting interaction between glycoprotein IIb/IIIa receptor on platelets
465. The normal therapeutic dose of aspirin can:
a) Inhibit uric acid synthesis
b) Inhibit leukotriene synthesis
c) Produce greater anti-inflammatory effect as compared to acetaminophen
d) Less effective for headache than acetaminophen
e) Inhibit the growth of fever causing bacteria
466. Which one of the following drugs non selectively inhibits both COX-I and COX-2?
a) Celecoxib
b) Aspirin
c) Acetaminophen
d) Zileuton
e) Zafirleukast

467. 5 lipoxygenase used in lipoxygenase pathway synthesises:


a) Prostaglandin
b) Prostacycline
c) Thromboxanes
d) Platelet activating factor (PAF)
e) Leukotrienes
ANTIBACTERIAL DRUGS:
468. Chemotherapy implies:
a) Treatment of specific infections by chemical action in the body of the patient.
b) Destroying or removing the bacteria from the body by destroying all the bacteria
c) Systemically treating the specific infections without serious harm to the host.
d) Systemically inhibiting both the growth and multiplication of bacteria
e) Acting systemically to kill all the bacteria, fungi, viruses and Parasites
469. Antibiotics are:
a) The chemical preparations used to kill the bacteria in the body.
b) The synthetic compounds used to treat infections
c) The Pharmaceutical preparations used for removing the infective agent
d) The drugs used against bacteria, viruses ,fungi and parasites
e) The drugs produced by microorganisms and used against microorganisms.
470. The commonest type of transfer of DNA material of resistance from one bacterium to another is:
a) Expression of proteins of resistance
b) Plasmid mediated
c) Chromosomal
d) Antibiotic inactivating enzymes
e) P- glucoprotein efflux pump
471. A bacteriostatic drug is not given with a bactericidal drug because:
a) The bacteriostatic drug may not find any living bacteria for its action
b) The bacteriostatic drug has a comparatively weaker action of inhibition .
c) Bactericidal drug will hardly need inhibition of bacteria for its killing
d) It will expose the patient to the adverse effects of two antibiotics
e) It can even stop the action of both ,one acting on active cells and other inhibiting them.

472. When two bacteria come in contact with each other and genetic material in the form of plasmid is
transferred from the resistant one to the non resistant one the mechanism is called:
a) Transduction
b) Transformation
c) Conjugation
d) Transposition
e) Insertion

473.Sites of action of antibacterial agents include all of the following except:

a) DNA gyrase

b) Microtubules

c) 50 s and 30 s ribosomal submits

d) Cell wall synthesis

e) RNA polymerase

BETA LACTAM ANTIBIOTICS:

474. Which of the following drugs is effective against penicillinase resistant staphylococcus aureus?
a) Carbenicillin
b) Ticarcillin
c) Oxacillin
d) Ampicillin
e) Mezlocillin
474-2 Beta-lactam antibiotics are bactericidal due to their interference with which of the following

processes?

a. N-acetylation of glucosamine

b. N-acetylation of muramic acid

c. Polymerization of monosaccharides

d. Ribosomal protein biosynthesis

e. Transpeptidation reaction

475. Activity of penicillins is due to :


a) Thiozolidine ring
b) Amino group
c) β- lactam ring
d) Acidic group
e) Double bond

476. All exept the following penicillins are excreted in urine:


a) Penicillin-G
b) Penicillin-V
c) Ampicillin
d) Amoxicillin
e) Nafcillin

477. Penicillin exerts its antibacterial effect by:


a) Changes in penicillin binding proteins
b) Producing β-lactamases
c) Producing peptidoglycan
d) Inhibiting autolytic enzymes
e) Inhibiting transpeptidases

478. All of the below are anti pseudomonal penicillins exept:


a) Methicillin
b) Carbenicillin
c) Ticarcillin
d) Azlocillin
e) Mezlocillin

479. All of below are penicillinase resistant penicillins except:


a) Oxacillin
b) Methicillin
c) Cloxacillin
d) Ampicillin
e) Dicloxacillin

480. Which of the following drugs can cause flushing of the body called “red man syndrome” due to histamine
release?
a) Penicillin G
b) Azithromycin
c) Doxycycline
d) Vancomycin
e) Gentamicin

481. Which one of the following choice is appropriate for the treatment of clostridium difficile infection?
a) Ampicillin
b) Clarithromycin
c) Trimethprim plus sulfamethoxazole
d) Gentamicin plus lincomycin
e) Metronidazole plus vancomycin
482. Which one of following drugs is beta lactam antimicrobial?

a) Sulphadiazine

b) Meropenem

c) Clotrimazole

d) Neomycin

e) Ciprofloxacin

483.Which one of the following statement about ampicillin is incorrect?

a) Its activity is enhance by sulbactam

b) It is the drug of choice for L monocytogenes infection

c) It causes maculopapular rashes

d) It eradicates most strains of MRSA

e) Pseudomembranous colitis may occur with its use

484. Administration of cefmetazole, cefotetan and cefoperazone is associated with higher incidence of?
a) Ototoxicity
b) Hypertension
c) Congestive cardiac failure
d) Bleeding tendency in patients on warfarin
e) Renal toxicity

485. Out of pencillins, ticarcillin has a greater potential of:


a) Producing β lactamase dependant resistance
b) Causing bleeding, hypevolemia and hypertension
c) Causing acute renal failure
d) Producing bronchoconstriction
e) Causing lupus like syndrome

486. Which of the following properties is common with β-lactams and amphotericin B.
a) Agranulocytosis
b) Not given to the patient of AIDS
c) Act on cell wall or cell membrane
d) Enzyme inducers
e) Leukocytosis

487. Which of the following antibiotics is not given to an alcoholic specially when he has recently drunk:
a) Cefamandole
b) Ampicillin
c) Azithromycin
d) Benzathine penicillin
e) Doxycycline

488. The poor activity of narrow spectrum penicillins against gram negative bacteria is due to the fact that:
a) Gram negative bacteria can metabolically inactivate narrow spectrum penicillins
b) These cause active efflux of penicillin
c) These bacteria have no penicillin binding proteins
d) Gram negative bacteria have no activating enzymes
e) Gram negative bacteria have an outer membrane with very small pores that do not allow entry
of penicillin into the bacilli

489. First generation cephalosporins in addition to penicillin like spectrum also include:
a) Hemophilus, enterobacteraceae and neiseria species
b) Staph aureus,streptococcus ,and E coli
c) Ecoli, Neiseria and Staphylococcus aureus
d) Proteus, E coli and Klebsiella
e) Pseudomonas, E coli and Klebsiella

490.Second generation cephalosporins also include:


a) Providensia, Ecoli and Klebsiella
b) Hemophilus, Enterobacter and Neiseria
c) Proteus, Enterobacter and Klebsiella
d) Hemophilus, Ecoli and Neisseria
e) Proteus,E coli and Hemophilus

491. Third generation cephalosporins have:


a) An additional action on Staph cocci, E coli and Pseudomonas aerogenosa
b) An additional action against gram positive bacteria
c) An additional action against streptococci, Ecoli and proteus
d) More action against gram negative bacteria than gram positive bacteria
e) More action against Staph aureus,E coli, Pneumococci and Pseudomonas
492. All of the following have a methylthiotetrazole nucleus and may cause alcoholic intolerance and bleeding
except:
a) Cafotaxime
b) Cafotetan
c) Cafomandole
d) Cafoparazone
e) Moxolactam

493. Fourth generation cephalosporins have special activity against otherwise resistant:
a) Serratia marcessens
b) Providensia
c) Citrobacter
d) Bacteriodes fragilis
e) Enterobacteriaceae
494. Which one of the following cephalosporins is excellently effective against bacteroides fragilis?
a) Cephapirin
b) Cephalothin
c) Cefoxitin
d) Cefaclor
e) Cefuroxime
495. A pregnant woman was hospitalized and catheterized with a foley,s catheter. She developed a urinary

tract infection caused by pseudomonas aeruginosa and was treated with gentamicin. Which of the following

adverse effects was a risk to the fetus when the woman was on gentamicin?

a) Skeletal deformity

b) Hearing loss

c) Teratogenesis

d) Blindness

e) Mental retardation

496. Which of the following cephalosporin‟s can be used in meningitis ?

a) Cefoperazone

b) Cefixime

c) Ceftibuten

d) Cefuroxime

e) None of above

497. All of the following antimicrobial drugs are bacteriostatic except:

a) Clindamycin

b) Erythromycin

c) Tetracycline

d) Trimethoprim

e) Vancomycin

CHLORAMPHENICOL:
498. Chloramphenicol is an effective antibiotic but significant toxicity limits its use particularly in newborns
and infants. Which of the following is the major and most common toxic reaction of this drug:
a) Aplastic anemia

b) Hepatotoxicity

c) Interstitial nephritis

d) Pulmonary fibrosis

e) Torsades de points or ventricular fibrillation


499. The dose of Chloramhenicol is reduced in:
a) CNS infections like meningitis
b) Enteric fever
c) Liver disease
d) GIT upset
e) Kidney disease

500. Chloramphenicol acts by:


a) Activating chloramphenicol transferase
b) Inhibiting chloramphenicol transferases
c) Binding to the transfer RNA
d) Binding to 30S subunit of bacterial ribosome
e) Binding to 50S subunit of bacterial ribosome

501. Resistance against chloramphenicol develops by:


a) Change in permeability of bacterial cell wall to the drug
b) P-glycoprotein efflux pump
c) Inhibition of transpeptidases
d) Production of Chloramphenicol acetyl transferase enzyme.
e) Production of peptidyl transferase enzyme.

502. The spectrum of chloramphenicol does not include:


a) Hemophilus influenza
b) Anerobic bacteria
c) Candida infection
d) Neisseria meningitides
e) Recketsial nfections

503.Gray baby syndrome develops because of:


a) Production of glucoronic acid
b) Underproduction of glucoronic acid
c) Over production of peptidyl dipeptase
d) Overproduction of glucoronyl transferase
e) Poor glucoronyl transferase conjugation system

TETRACYCLINES:
504. Which one of the following is a long acting tetracycline?
a) Chlortetracycline
b) Oxytetracycline
c) Doxycycline
d) Demeclocycline
e) Methacycline
504-2. The following antibiotic can be prescribed safely in patients having renal impairment?

a. Amikacin

b. Vancomycin

c. Gentamycin

d. Doxycycline
e. Cephazolin

505. Which one of the following tetracyclines can be given to a patient with compromised renal function?
a) Oxytetracycline
b) Doxycycline
c) Chlortetracycline
d) Methacycline
e) Demeclocycline

506. Which one of the following is effective against chlamydia, mycoplasma and rickettsia?
a) Doxycycline
b) Bacitracin
c) Vancomycin
d) Gentamicin
e) Penicillin G

507. Tetracycline given with antacids cause drug interaction because:


a) Antacids form a protective layer which hampers absorption
b) Antacids increase the pH of the stomach
c) Tetracycline kills the intestinal flora
d) Both antacids and tetracycline denanature the enzymes
e) Teracycline forms insoluble compounds with antacids

508. Which one of the following drugs may end in failure of therapy if it is always taken with milk?
a) Pencillin V
b) Doxycycline
c) Neomycin
d) Isoniazid
e) Cefixime

509.Regarding tetracycline which one of the following is incorrect?

a) Inhibit protein synthesis in bacteria

b) Better absorption after meals

c) Are bacteriostatic

d) Can cause phototoxicity

e) Most of them are excreted by glomerular filtration

510. The half life of long acting tetracycline is:


a) 48 to 72 hours
b) 24 to 48 hours
c) 24-to30 hours
d) 16 to 20 hours
e) 6 to 9 hours

MACROLIDES:
511. A 35 year old pregnant lady with a history of pyelonephritis has developed severe upper respiratory

tract infection that appears to be due to bacterial pathogen Which antibacterial agent will be safe for the

treatment of infection in this pregnant lady?

a) Clarithromycin

b) Streptomycin

c) Azithromycin

d) Tetracycline

e) Quinolones

512. Which of the following drugs is approved for use in vancomycin resistant enterococcal infections?

a) Clarithromycin

b) Erythromycin

c) Linezolid

d) Minocycline

e) Ticarcillin

513.A 25 years old male patient develops antibiotic associated pseudomembranous colitis in response to drug

therapy. Which of the following was the most likely cause of this severe problem?

a) Azithromycin

b) Amoxicillin

c) Clindamycin

d) Metronidazole

e) Trimethoprim plus sulfamethoxizole

514. Which advantage makes the amikacin more effective against pseudomonas aeroginosa infection as
compared to gentamicin?
a) It is less ototoxic
b) No need of blood level monitoring
c) No nephrotoxicity
d) It is also effective against anaerobes
e) It has broadest spectrum against gram negative bacilli

AMINOGLYCOSIDES:

515. Aminoglycosides are:


a) Actively transported across the outer cell membrane
b) Cell wall inhibitors
c) More active against gram positive organisms
d) Highly lipid soluble compounds
e) Actively transported across the inner cell membrane

516. Aminoglucosides bind to the:


a) 50S subunit of bacterial ribosome
b) 30 S subunit of bacterial ribosome
c) Bacterial transfer RNA
d) Growing peptide chain
e) Amino acid to prevent it from addition into peptide chain
517. Amino glycoside acts synergistically with penicillin because:
a) Aminoglucosides and penicillins both effectively kill gram positive bacteria
b) Combined together these kill the anerobic bacteria
c) Penicillin damages the bacterial wall to facilitate the aminoglycoside entery.
d) Penicillin and aminoglycosides open up the outer membranes pores
e) Aminoglucoside inactivate beta lactamases

518. Aminglucosides are indicated in all of the following diseases except:


a) Nephritis
b) Tuberculosis
c) Plague
d) Tularemia
e) Endocarditis by strptococus viridians

519. Aminoglycosides can cause the following adverse effects except:


a) Optic nerve damage
b) Damage to the brain stem
c) Vastibular damage
d) Neuro muscular block
e) Hemolytic anemia

520. In adding penicillin with gentamicin:


a) Increases the risk of resistance
b) Gentamicin will inactivate penicillin
c) Gentamicin will potentiate penicillin
d) Penicillin and gentamicin will increase nephrotoxicity
e) The patient will develop pseudomembranous colitis

521. Gentamicin inhibits bacterial protein synthesis by binding to: -


a) DNA
b) mRNA
c) RNA polymerase
d) 30 S ribosomal subunit
e) Peptidoglycan in the cell wall

522.Which of the following is bacteriostatic?

a) Penicillin

b) Gentamicin

c) Vancomycin

d) Cefuroxime

e) Chloramphenicol

523. Which of the following is most effective against H. influenzae meningitis?


a) Penicillin V
b) Azithromycin
c) Benzathine penicillin
d) Ceftriaxone
e) Procaine penicillin
524. Absortion of tetracycline is impaired by milk because of:
a) Casein layer
b) Lactose
c) Ca and other chelates
d) Alkaline pH created
e) Competitive inhibition

525. Tetracycline acts by:


a) Binding to 30 S ribosomal subunit of bacteria
b) Binding to 50 S ribosomal subunit of bacteria
c) Binding to the transfer RNA of bacteria
d) (D)Binding to the growing peptide chain
e) Binding to the amino acyl RNA
526. Which of the following drugs (other than trimethoprim – sulfamethoxazole) is used in pneumocystis
carinii infection?
a) Penicillin G
b) Pentamidine
c) Nifurtimox
d) Carbenicillin
e) Metronidazole
AMINOGLYCOSIDES:
527. Which one of the following protein synthesis inhibitors is bactericidal and not bacteroistatic?
a) Doxycycline
b) Clarithromycin
c) Linezolid
d) Clindamycin
e) Gentamycin

528. Which one of the following can cause hearing loss?


a) Penicillin
b) Gentamicin
c) Cephalothin
d) Cefotetan
e) Ofloxacin

529. Which of the following antibiotics will prevent production and absorption of ammonia from G.I.T in
hepatic coma?
a) Doxycycline
b) Penicillin
c) Cephalexin
d) Clarithromycin
e) Neomycin
530. Anaerobic bacterial infection is unlikely to respond to gentamicin because:
a) Gentamicin itself synthesises resistance factors against its own antianaerobic action
b) Anaerobes cannot activate aminoglycosides
c) Anaerobes cannot metabolize gentamicin
d) Anaerobes lack O2 necessary for binding to 50 S subunit
e) Without oxygen gentamicin cannot be transported across the bacterial cell membrane
SULFONAMIDES,TRIMETHOPRIM:
531. Sulfonamide given to newborn may cause kernicterus because:
a) Sulfonamides cause directly damage to basal ganglia
b) New born has no enzymes to metabolize Sulfonamide
c) Sulfonamides displace bilirubin from their binding proteins
d) Sulfonamides increase the bilirubin synthesis
e) Sulfonamide decrease the excretion of bilirubin

532.A 30 years old man who was suffering from AIDS, suffered from pneumonia due to pneumocystis

jirovecie organism, which of the following drugs may be used in this patient for treatment of pneumonia?

a) Erythromycin

b) Chloroquine

c) Streptomycin

d) Co-trimaxozole

e) Ampicillin

533. Sulfonamides increase the risk of neonatal kernicterus because they:

a) Diminish the production of plasma albumin

b) Increase the turnover of red blood cell

c) Inhibit the metabolism of bilirubin

d) Compete for bilirubin binding sites on plasma albumin

e) Depress the bone marrow

QUINOLONES:

534-1. Out of the following quinolones, which one is most effective against pseudomonas aeroginosa infection?
a) Ofloxacin
b) Pefloxacin
c) Ciprofloxacin
d) Lomefloxacin
e) Enoxacin
1) 534-2. The following is the mechanism of action of Flouroquinolones.

a. Inhibit microbial cell wall

b. Inhibit microbial folic acid synthesis

c. Inhibit synthesis of microbial DNA

d. Inhibit microbial protein synthesis

e. None of them

535. Which fluoroquinlone is also active against anaerobic bacteria?


a) Moxifloxacin
b) Gemifloxacin
c) Gatifloxacin
d) Ofloxacin
e) Norfloxacin
536. Which one of the following fluoroquinolones has a better activity profile against gram negative
organisms?
a) Nor floxacin
b) Gatifloxacin
c) Ciprofloxacin
d) Gemifloxacin
e) Moxifloxacin
537. Which one of the following is a first generation fluoroquinolone?
a) Ciprofloxacin
b) Levofloxacin
c) Norfloxacin
d) Oxolinic acid
e) Cinoxacin
538. Which one of the following is a third generation fluoroquinolone?
a) Levofloxacin
b) Lomefloxacin
c) Ciprofloxacin
d) Moxifloxacin
e) Eoxacin

539. Which one of the following older quinolone does not achieve systemic anti bacterial levels?
a) Moxifloxacin
b) Gatifloxacin
c) Levofloxacin
d) Ciprofloxacin
e) Nalidixic acid
ANTI MYCOBACTERIAL DRUGS:
540. Arabinoglycan, an essential component of mycobacterial cell wall is inhibited by which one of the
following drugs?
a) Streptomycin
b) Isoniazid
c) Ethambutol
d) Rifampin
e) Pyrazinamide

541. An essential component of mycobacterial cell wall is mycolic acid. Its synthesis is inhibited by which one
of the following anti TB drugs?
a) Ethambutol
b) Streptomycin
c) Isoniazid
d) Pyrazinamide
e) Rifampin

542. Which of the following acts against tubercle bacilli inside the macrophages?
a) Pyrazinamide
b) Isoniazid
c) Ethambutol
d) Streptomycin
e) Rifampin

543. Which serious adverse effect is associated with ethambutol?


a) Cholestatic jaundice
b) Renal dysfunction
c) Myelosuppression
d) Retrobulbar neuritis
e) Psychosis

544. Which of the drug regimens is recommended for treatment of latent TB?

a) Rifampin for 6 months

b) Isoniazid for 9 months

c) Isoniazid and rifampin for 6 months

d) Ethambutol for 6 months

e) None of the above

545. A patient started on 1st line anti TB drug presents in the OPD with the complaint of reduced visual

acuity after 3 month. Which drug has to be stopped?

a) Streptomycin

b) Rifampicin

c) Pyrazinamide

d) Ethambutol

e) Isoniazid

546. Resistance against rifampin develops in mycobacteria mainly by:


a) Point mutations which prevent binding of the drug to RNA polymerase
b) Mutations causing over expression of RNA polymerase
c) Decreased penetration of the drug into the mycobacteria
d) Increased synthesis of rifampin inactivating enzyme
e) Increased activity of P-glycoprotein efflux pump which expels the drug out

547. Which of the following drugs; is given against the neuropathy developing due to isoniazid?
a) Thiamine
b) Niacin
c) Pyridoxine
d) Cyanocobalamine
e) Ascarbic acid

548. Out of the following, which one is a second line antituberculous drug?
a) Pyrazinamide
b) Ciprofloxacin
c) Isoniazid
d) Rifampin
e) Ethambutol
549. Which one of the following antituberculous drugs is an enzyme inducer and may cause drug interactions?
a) Isoniazid
b) Ethambutol
c) Streptomycin
d) Rifampin
e) Pyrazinamide

550. Which of the following antituberculous drugs can cause permanent staining of contact lenses?
a) Streptomycin
b) Rifampin
c) Pyrazinamide
d) Isoniazid
e) Ethambutol

551. A 40 year old man has been on primary therapy for active pulmonary tuberculosis for the past 2

months. At his regular clinic visit, he complains of a pains and needles sensation in his feet. You suspect that

he might be deficient in which one of the following vitamins?

a) Ascorbic acid

b) Niacin

c) Pyridoxine

d) Calcitriol

e) Folic acid

552.Mycolic acid synthesis is inhibited by which one of the following?

a) Ethmobutol

b) Isoniazid

c) Pyrizinamide

d) Rifampin

e) Streptomycin

553.A 45 years old man with history of gout is diagnosed to be having tuberculosis regarding selection of

drugs for the treatment of tuberculosis which drug should be avoided?

a) Rifampicin

b) Isoniazid

c) Streptomycin

d) Ethambutol

e) Pyrazinamide

554. All of the following benefits of isoniazid are valid except?


a) It is bactericidal
b) It is effective against intracellular bacilli
c) It is effective when given orally
d) D. It is equally effective against the dormant and rapidly multiplying tubercle bacilli
e) It can easily reach CSF
555. Which of the following two drugs are effective against both tuberculosis and leprosy?
a) Pyrazinamide and PAS
b) Amikacin and Rifampin
c) Capreomycin and isoniazid
d) Rifampin and clofazimine
e) Ethambutol and isoniazid

556. All of the following are the first line antibubercular drugs except?
a) Rifampin
b) Rifabutin
c) Isoniazid
d) Pyrazinamide
e) Ethambutol
ANTIFUNGAL DRUGS
557. What are the potential adverse effects of amphotericin B?
a) Bone marrow suppression
b) Gastric hyperacidity
c) Hypercholestrolemia
d) Acute liver failure
e) Fever and hypotension ( release of TNF-α and IL-1)

558. Which one of the flollowing is the mechanism of action of turbinafine?


a) It inhibits ribrosomal protein synthesis
b) It inhibits squalene epoxidase
c) It forms pores in the cell membrane
d) It inhibits DNA synthesis
e) It inhibits mixed function oxidases

559. Antifungal activity of flucytosine is explained by which one of the following statements?
a) It inhibits DNA and RNA synthesis
b) It inhibits squalene epoxidase
c) It forms pores in the cell membrane
d) It inhibits cytochrome P450 enzymes
e) It inhibits protein synthesis by ribosomes

560. Amphotericin B‟s mechanism of action is explained best by which statement of the following?
a) Formation of pores in the cell membrane
b) Blocks the squalene epoxidase
c) Inhibits mixed function oxidases
d) Blocks DNA and RNA synthesis
e) Inhibits protein synthesis by ribosomes

561. The most serious adverse effect of amphotericin B is:


a) Liver dysfunction
b) Kidney dysfunction
c) Pancytopenia
d) Neuropathy
e) Otoxicity
562. Which one is orally effective antifungal drug for cryptococcal meningitis?
a) Nystatin
b) Amphotericin B
c) Fluconazole
d) Ketoconazole
e) Metronidazole

563. Which one of the following antifungal drugs is effective agent for onychomycosis?
a) Miconazole
b) Terbinafine
c) Clotramizole
d) Griseofulvin
e) Caspofungin

564. Which one of the following drugs is used for mucocutaneous candidiasis?
A) Fluconazole
b) Griseofulvin
c) Flucytosine
d) Ketoconazole
e) Voriconazole
565. Which one of the following is an adverse effect of ketoconazole?
a) Sebrrhic dermatitis
b) Adrenal insufficiency
c) Inhibition of estrogen synthesis
d) Aseptic meningitis
e) Achlorhydria
566. Which laboratory test should be monitored while using griseofulvin?
a) Bicarbonate
b) Chloride
c) Liver function tests
d) Calcium
e) Creatinine
567. Which one of the following combination of drugs is most appropriate for treating cryptococcal
meningitis?
a) Clotrimazole + nystatin
b) Itraconazole + nystatin
c) Amphotericin B + flucytosine
d) Clotrimazole + flucytosine
e) Caspofungin + Fluconazole

568. For a life threatening fungal infection which one of the following should be chosen for the antifungal
therapy?
a. Caspofungin
b. Amphotericin B
c. Fluconazole
d. Itraconazole
e. Flucytosine
569. Which one of the following antifungal drugs is delivered with a lipid vehicle to prevent non specific
binding of the drug to the patients cells to reduce the chances of renal cell damage?
a) Ketoconazole
b) Amphotericn B
c) Terbinafine
d) Flucytosine
e) Itraconazole

570. Which of the following is used for cutaneous dermatophyte infection?

a) Griseofulvin

b) Bacitracin

c) Neomycin

d) Polymyxin

e) Penciclovi

ANTIVIRAL DRUGS:

571. Administration of saquinavir with ritonavir is justified by which one of the following statements?
a) To reduce the toxicity of saquinavir
b) To reduce the gastrointestinal side effects
c) To reduce the chances of resistance development
d) Ritonavir inhibits cytochrome P450 to increase the concentration of saquinavir despite its first
pass effect
e) To potentiate the effect of saquinavir

572. Didanosine acts by which one of the following mechanisms?


a) It inhibits uncoating of viral RNA
b) It inhibits viral reverse transcriptase
c) It inhibits protein synthesis
d) It inhibits RNA synthesis
e) It inhibits the packaging and assembly of virion

573. For inhibition of influenza A virus, the drugs amantadine and rimantadine act by which one of the
following mechanisms?
a) Target the M2 protein within the membrane and inhibit uncoating of virion
b) Inhibit DNA synthesis
c) Inhibit reverse transcriptase
d) Inhibit protein synthesis
e) Inhibit the packaging and assembly of the virion

574. Out of the following protease inhibitors, the one with highest degree of CSF penetration is:
a) Saquinavir
b) Ritonavir
c) Indinavir
d) Amprenavir
e) Nelfinavir
575. The major problem with ritonavir is:
a) Low bioavailability
b) It is an inhibitor of CYP450 and thus causes drug interactions
c) It is an inducer of CYP450 enzymes
d) Causes pancreatitis
e) It is nephrotoxic
576. A common adverse effect of zalcitabine is:
a) Myelosuppression
b) Liver dysfunction
c) Peripheral neuropathy
d) Pancreatitis
e) Renal toxicity
577. Probenecid is given with Zidovudine :
a) To facilitate its excretion
b) To decrease its excretion
c) To increase its absorption
d) To increase its biotransformation
e) To avoid its toxicity
578. The following is a drug of choice for treating cytomegalovirus infection in AIDS patient.

a) Limivudine

b) Ganciclovir

c) Didanosine

d) Acyclovir

e) Zidovudine

579. Which one is a risk associated with the use of didanosine?


a) Hepatitis
b) Pancytopenia
c) Renal impairment
d) Pancreatitis
e) Cardiomyopathy

580. Zidovudine is:


a) A protease inhibitor
b) A nucleoside reverse transcriptase inhibitor
c) An anticytomegalovirus drug
d) Nucleotide reverse transcriptase inhibitor
e) Non nucleoside reverse transcriptase inhibitor

581. Administering cidofovir intravenously is associated with which adverse effect?


a) Nephrotoxicity due to renal secretion
b) Hepatotoxicity
c) Ototoxicity
d) Skin rash
e) CNS toxicity

582. Which one is the common drug interaction when zidovudine and ganciclovir are concurrently
administered?
a) Ganciclovir induces CYP450 enzymes and decreases the serum level of zidovudine
b) Myelosuppression due to additive effects
c) CNS toxicity
d) Ganciclovir inhibits CYP450 enzymes and increases the toxicity of zidovudine
e) Ganciclovir decreases the biliary excretion of zidovudine

583. The resistance against acyclovir develops by:


a) Alteration of reverse transcriptase
b) Alteration in proteases
c) Alteration of neuraminidase
d) P-glycoprotein efflux pump
e) Alteration in thymidine kinase / DNA polymerase

584. Acyclovir acts by:


a) RNA synthesis
b) Uncoating of viral DNA
c) Incorporation into viral DNA
d) Packaging of virion
e) Assembly of virion

ANTI PROTOZOAL DRUGS:

585. Which one of the following antimalarial drugs inhibits dihydrofolate reductase of the malarial parasite?
a) Chloroquine
b) Quinine
c) Amodiaquine
d) Artemisinin
e) Pyrimethamine

586. For eradication of the dormant hepatic stage ,which one out of the following drugs is given for
plasmodium vivax and plasmodium ovale?
a) Primaquine
b) Amodiaquine
c) Quinine
d) Mefloquine
e) Chloroquine

587.Which of following antimalarial drugs is effective in eradication of p. vivax secondary exoerythrocytic

phase?

a) Chloroguanide

b) Chloroquine

c) Primaquine

d) Pyrimethamine

e) Mefloquine

588. Which of the following drugs is correctly indicated for extra intestinal amoebiasis?
a) Paromomycin
b) Metronidazole
c) Dialoxanide furoate
d) Mefloquine
e) Iodoquinol
588-2. Metronidazole is not effective in against the following organism.

a. Bacteroides fragilis

b. Helicobacter pylori

c. Clostridium difficile

d. Pseudomonas aeruginosa

e. Clostridium tetani

589. A 30 year old patient with G6PD deficiency after the use of anti-malarial has dark colored urine. Which

of the anti-malaria has he used?

a) Pyrimethmine

b) Artemisinin

c) Chloroquine

d) Primaquine

e) Doxycycline

590. Out of the following ,for which infection tetracycline is an appropriate drug?
a) Herpes zoster
b) Rickettsias
c) Plasmodium vivax
d) Shistosoma hematobium
e) Histoplasma

591. Which one of the following combination can treat both erythrocytic and exoerythrocytic stages of
malarial parasite?
a) Piperazine and chloroquine
b) Chloroquine and metronidazole
c) Sulfadoxine and trimethoprim
d) Primaquine and chloroquine
e) Amodiaquine and chloroquine

592. A trader along with his team is planning to take his goods to an area where multi-drug resistant

malaria is common. Which drug would you prescribed prophylactically against this condition?

a) Primaquine

b) Choloroquine

c) Malaron (Atovaquone + proguanil)

d) Mefloquine

e) Doxycycline
ANTHELMINTICS:

593. Which one of the following is broad spectrum anthelmintic drug?


a) Piperazine
b) Bithionol
c) Albendazole
d) Metronidazole
e) Oxamniquine

594. Which one of the following is the drug of choice in hydatid cyst?
a) Ivermectin
b) Piperazine
c) Albendazole
d) Praziquantel
e) Niclosamide

595. Which one of the following is the drug of choice for tape worm infestation?
a) Albendazole
b) Mebendazole
c) Piperazine
d) Metronidazole
e) Praziquantel

596. Which one of the following is the drug of choice for Dracunculus medinensis (Guinea worm) infestation?
a) Metronidazole
b) Ivermectin
c) Niclosamide
d) Thiabendazole
e) Piperazine

597. Which one of the following is the drug of choice for filariasis?
a) Niclosamide
b) Pyrental pamoate
c) Mebendazole
d) Diethylcarbamazine
e) Piperazine

598. Which one of the following is the drug of choice for strongyloides stercoralis (thread worm)?
a) Ivermectin
b) Bithional
c) Thiabendazole
d) Metronidazole
e) Tinidazole

599. Which one of the following is the drug of choice for hook worm infestation?
a) Bithionol
b) Piperazine
c) Pyrental pamoate
d) Ivermectin
e) Metronidazole

600. Which one of the following is the drug of choice for the round worm?
a) Niclosamide
b) Piperazine
c) Praziquantel
d) Albendazole
e) Metronidazole

ANTI CANCER DRUGS:

601Vinca alkaloids act at which phase of cell cycle ?

a) G1 phase

b) G2 phase

c) G1 and G2 phase

d) M phase

e) S phase

602. Which one of the following anticancer drugs intercalates between the base pairs of DNA and the DNA
dependant RNA synthesis is inhibited?

a) Paclitaxel

b) Docetaxel

c) Dactinomycin

d) Vincristine

e) Methotrexate

603. Which one of the cell cycle specific drugs inhibits topoisomerase II enzyme?

a) Teniposide

b) Cyclophosphamide

c) Methotrexate

d) Chlorambucil

e) Dacarbazine

604. Which of the following cell cycle specific anticancer drugs acts by arresting metaphase?

a) Vinblastine

b) Doxorubicin

c) Daunorubicin

d) Carboplatin

e) Fluorouracil
605.Tamoxifen is a relative estrogen receptor modulator that acts therapeutically as an estrogen in

bone and as an estrogen antagonist in:

a) Liver

b) Breast

c) Uterus

d) Ovary

e) Brain

606. DNA polymerase, an enzyme essential for the DNA synthesis is inhibited competitively by a drug
metabolite. Which one of the following is the parent drug?

a) Docetaxel

b) Cytarabine

c) Chlorambucil

d) Teniposide

e) Mechlorethamine

607. Which one of the following drugs causes alkylation of the groups like:

i. Carboxyl

ii. Hydroxyl

iii. Amino

iv. Sulfhydryl

v. Phosphate
And alkylation of DNA which causes cell death

a) Vincristine

b) Vinblastine

c) Cyclophosphamide

d) Daunorubicin

e) Methotrexate

608. An intermediate compound polyglutamate is responsible for the formation of anticancer derivative.
Which one is the parent drug?

a) Vincristine

b) Cytarabine

c) 5 fluorouracil

d) 6 mercaptopurine

e) Methotrexate
609.A 35 year old man has recently taken treatment for a malignancy. He now presents to emergency with

chief complaint of shortness of breath. On investigation he was found to be having cardiomyopathy. Which

drug is prone to cause this side effect?


a) Vincristine

b) Bleomycin

c) Doxorubicin

d) Asparagine‟s

e) None of them

610. Which one of the drugs is known to be a cell cycle specific anticancer drug?

a) Vincristine

b) Cyclophosphamide

c) Mitomycin

d) Carboplatin

e) Streptozocin
611. In which one of the following breast carcinoma stages ,the chemotherapy as adjuvent to surgery is
regarded most beneficial?

a) Stage I

b) Stage II with one or two lymph nodes involved

c) Stage II with four or five lymph nodes involved

d) Stage III

e) Stage IV

612. Which one of the following side effects is commonly associated with the use of alkylating agents?

a) Decreasing RBCs

b) Decreasing WBCs

c) Liver dysfunction

d) Renal insufficiency

e) Peripheral neuropathy

613. Which of the following drug is cell cycle non specific anticancer drug?

a) Methotrexate

b) Vinblastine

c) Cyclophosphamide

d) Etoposide

e) Bleomycin

614. Which one of the following exhibits minimal if any cross resistance with other given drugs?

a) Vincristine

b) Dactinomycin

c) Bleomycin

d) 6-mercaptopurine
e) Doxorubicin

615. Which one of the following anticancer drugs can induce antibodies against itself?

a) Cisplatin

b) Methotrexate

c) Cytosine

d) Carboplatin

e) Asparaginase

616. Which of the following drug is G2 phase specific?

a) Cisplastin

b) Paxitaxel

c) Vineristine

d) Methotraxate

e) Mitomycin

617. Which of the following anticancer drugs can be both ototoxic and nephrotoxic?

a) Mechlorethamine

b) Vincristine

c) Chlorambucil

d) 5-fluorouracil

e) Cisplatin

618. Any anticancer drug is explained by which of the following justifications?

a) Anticancer drug kills a constant number and not a constant fraction of cells

b) The dose response curve is shallow

c) The concentration and time determine the magnitude of cell kill

d) The dose limiting toxicity of a combination should be same and not a variety of different
toxicities

e) The therapeutic index is always wide

619. An old man with carcinoma prostate is being treated with leuprolide. Which other drug must also be
given adjunctively?

a) Testosterone

b) Letrozole

c) Flutamide

d) Prednisone

e) Tamoxifen

620. Why chemotherapy becomes less effective when the cancer has advanced?
a) P-glycoprotein becomes less active

b) The growing cells fraction slows and the cells in Go phase increase

c) Advanced tumour consumes the drugs rapidly

d) Topoisomerase becomes more active

e) With growing tumor the metabolism also increases and then overcomes the drug more easily.

621. Which property is associated with imatinib used as an anti myelogenous leukemia drug?

a) Intravascular clotting risk due to thrombocytosis

b) An ineffective drug for CML

c) Increased diuresis causing hypovolemia and hypotension

d) It inhibits cytochrome P450 and the toxicities of other drugs metabolized by these enzymes are
increased

e) Inhibits tyrosine kinase


IMMUNOMODULATING DRUGS:

622. Which of the following antimicrobial drugs stimulates phagocytes and produces oxygen metabolites toxic
to the microorganism?

a) Interferon alpha 1a

b) Interferon beta 1b

c) Betamethasone

d) Prednisone

e) Interferon gamma 1b

623. Capillary leak syndrome is associated with which one of the following drugs?
a) Interferon alpha 1a
b) Interferon alpha 2b
c) Interferon gamma 1b
d) Levamisole
e) Human recombinant IL-2

624. Which of the following drugs restores the suppressed immune functions of
- B-lymphocytes
- T lymphocytes
- Monocytes
- Macrophages
a) Tacrolimus
b) Sirolimus
c) Azathioprine
d) Levamisole
e) Mycophenolate mofetil
625. Which one of the following is a non specific immunosuppressant drug?
a) OKT3
b) Mycophenolate mofetil
c) Methotrexate
d) Infliximab
e) Tacrolimus
626. W of the following is a protein derived vaccine?
a) L-Asparaginase ( purified from E coli used for ALL)
b) Secretin (a recombinant protein stimulates pancreatic secretions and gastrin)
c) OSpA ( a protein derived vaccine for Lyme disease)
d) Etanercept ( proteins fused together to use against anti inflammatory effects of TNF)
e) Purified protein derivative ( not vaccine but used for diagnosis of previous infection)

627. Which one of the following agents is an antitumor necrosis factor alpha chimeric monoclonal antibody?
a) Levamisole
b) Infliximab
c) Basiliximab
d) Daclizumab
e) Etanercept

628. Muromonab CD3 may cause cytokines release syndrome. Which of the following is administered prior to
the administration of muromunab CD3 to prevent its reaction?
a) Cetirizine
b) Chlorpheniramine
c) Basiliximab
d) Daclizumab
e) Glucocorticoids

629. Which of the following agents binds to FKBP-12, inhibiting calcineurin‟s phosphatase activity?
a) Tacrolimus
b) Azathioprine
c) Mycophenolate mofetil
d) Prednisolone
e) Basiliximab
630. Which of the following agents after forming complex causes inhibition of T lymphocytes activation and
proliferation of interleukin-2 receptors?
a) Mycophenolate mofetil
b) Basiliximab
c) Azathioprine
d) Tacrolimus
e) Sirolimus

631. Which of the following drug forms a complex with cyclophilin that is an inhibitor of T lymphocytes
calcineurin?
a) Sirolimus
b) Azathioprine
c) Cyclosporine
d) Basiliximab
e) Mycophenolate mofetil

632. Which one of the following immunomodulating agents should not be given during or expected pregnancy?
a) Levamisole
b) Thalidomide
c) Tacrolimus
d) Sirolimus
e) Cyclosporine

633. Which one of the following strategys‟ would help in a patient of status asthmaticus not responding to
turbutaline which induces severe tachycardia?
a) Isoprenaline administration
b) Nedocromyl administration
c) I/V corticosteroid
d) S/C epinephrine
e) I/M diphenhydramine

PROSTAGLANDINS:

634.Misoprostol, an analog of PGF is sometimes used adjunctive to stimulate gastric mucus production and

help to reduce the incidence of gastric ulcer associated with long term or high dose NSAIDs therapy for

arthritis. Which of the following is the other main use for this lipid derived autacoids?

a) Closure of a patent ductus arteriosus in newborns

b) Contraception in women who should not receive estrogens or progestin‟s

c) Induction of abortion in conjunction with mifepristone

d) Closure of a VSD in newborns

e) Suppression of uterine contractility in woman with premature labor

DRUGS USED FOR BRONCHIAL ASTHMA:

635. A patient of asthma gets worsening of his attacks with pollen allergy. He uses dimenhydrate but the
condition does not improve; rather it worsens until he switches over to the fexofenadine. What is the
underlying problem with H1 antagonist use?
a) It releases serotonin
b) It increases the metabolism of other anti asthma drugs
c) It interferes with the effects of endogenous epinephrine
d) It aggravates the problem by releasing acetylcholine which causes bronchoconstriction
e) It decreases the secretions and render the mucus thick and tenacious.

636. A patient with hypertension and edema is also suffering from asthma. He is advised furosemide and after
a couple of months, his respiratory difficulty further increases. What is the probable cause?
a) Furosemide releases bradykinin
b) Furosemide increase the excretion of the drugs for asthma
c) Furosemide dries the airways and increases viscosity of mucus
d) Furosemide blocks the effects of catecholamines
e) Furosemide causes bronchoconstriction

637. Montelukast acts successfully in certain asthmatics because:


a) It prevents antigen antibody reactions
b) It antagonizes the leukotrienes
c) It increases the release of epinephrine
d) It inhibits the phosphodiesterase which is the enzyme that degrades cAMP.
e) It increases the response of bronchiolar muscle to epinephrine

638. The drug ipratropium bromide provides relief in the patients of asthma and COPD because :
a) It inhibits inflammatory reaction
b) It prevents antigen antibody reaction
c) It increases the release of epinephrine
d) It blocks the endogenous bronchoconstrictor mediator
e) It inhibits phosphodiesterase
639.B₂-agonists have following pharmacological actions except:

a) Relaxation of bronchial smooth muscles

b) Inhibit release of mast cells and other inflammatory mediators

c) Increased heart rate

d) Decreased intracellular cAMP

e) Decrease K level

640. A 55 year old female who is taking propranolol for the management of cardiovascular disease

experiences acute asthmatic attacks, which of the following drugs would prescribe to attenuate this asthmatic

attack?

a) Cromolyn sodium

b) Salbutamol

c) Beclomethsone

d) Pratropium bromide

e) Formeterol

641. A patient with the history of asthmatic attacks has been given written instructions by his doctor that
whenever he needs analgesic, he should avoid aspirin and he may use acetaminophen. What can be the reason
for this caution?
a) In certain patients aspirin causes upregulation of muscarinic receptors in bronchial smooth
muscles.
b) It blocks the synthesis of prostaglandins and the leukotriene pathway is left open
c) It causes hypersensitivity of H1 receptors in the bronchial smooth muscles
d) It induces the synthesis of antibodies directed against mast cells of the bronchiolar smooth
muscles
e) It blocks the binding of epinephrine to β2 receptors

642. A 15 years old boy is given an antiasthmatic drug. He develops skeletal muscle tremors? Which of the
following may be responsible for the undesirable effect?
a) Nedocromyl
b) Montelukast
c) Salbutamol
d) Ipratropium bromide
e) Corticosteroid
643. An 21 years old girl is going well on salbutamol for her asthma. After a couple of months treatment
with it, she feels that it is no more sufficient to overcome her breathing difficulty. What strategy is now, most
appropriate for her?
a) Salbutamol should be replaced with turbutaline
b) Aminophylline should be given as an adjunct
c) Oral nedocromil should be given
d) Inhaled corticosteroid should be added
e) Oral prednisone should be added

644. A lethal dose of theophylline can cause death primarily due to:
a) Hepatic coma
b) Bradycardia
c) Kidney failure
d) Convulsions
e) Hypertensive crisis

645. In bronchial asthma theophylline is justified only for:


a) It‟s anti-inflammatory action
b) Prevention of symptoms
c) Being eliminated totally by the kidneys
d) It causing sufficient sedation as well
e) Having no drug interaction and is therefore chosen for empirical and combination therapy

646. Which one of the following adrenergic agonist drugs has no role in the management of bronchial asthma,
as it does not cause bronchodilation?
a) Norepinephrine
b) Epinephrine
c) Isoprenaline
d) Salmeterol
e) Terbutaline

647. A 20 years old boy using an antiasthmatic drug for his problem and getting a moderate but transient
degree of relief with it, visits another physician who tells him that no one has ever bothered to identify
his primary objective of pharmacological cure. What might be the missing goal?
a) Instant reversal of bronchospasm
b) Treat bronchial inflammation
c) Replace oral drugs with inhaled agents
d) Limit the frequency of inhalers use to reduce resistance
e) Change the antiasthmatic with a long acting β2 agonist

648. Which of the following drugs correctly matches with its antidote?

a) Lead- deferioxamine

b) TCA -flumazenil

c) Theophylline –β blockers

d) Warfarin-protamine

e) none of them
DRUGS ACTING ON G.I.T:

649. A patient after undergoing surgery received damage to the gastric innervations of vagus nerve ,causing
delay in gastric emptying. Which one of the following would be most helpful in facilitating gastric
motility?
a) Famotidine
b) Omeprazole
c) Domperidone
d) Hydroxyzine
e) Misoprostol
649-2. Regarding antiemetic drugs. Which of the following drug has anti-5HT-3. Anti-H1 and anti-D2

actions?

a. Ondansetron

b. Scopolamine

c. Domperidone

d. Prochlorperazine

e. Chlorpromazne

650. Generally the antiemetics are known to act primarily on:


a) H2 receptor in gastric mucosa
b) H1 receptor
c) Chemoreceptor trigger zone (CTZ)
d) Vomiting centre
e) 5HT receptors

651. Healing of a duodenal ulcer is primarily dependant upon:


a) Reducing the amount of histamine produced
b) Reducing the prostaglandin production
c) Restoration of the balance of motility by antacids
d) The protective layer of sucralfate
e) Reducing the nocturnal acid secretion

652. The action of lansoprazole depends upon:


a) Alkaline environment
b) Competitive inhibition of enzyme
c) Inhibition of H+/K+ adenosine triphosphatase
d) Giving it in combination with H2 receptor antagonist
e) Entry into the cell to act on the inside of cytoplasm

653. Which one of the patients should be specially warned against the use of antidiarrheal containing bismuth
subsalicylate?
a) With otitis media
b) With otitis externa
c) Under 12 years of age
d) With common cold, flu or chicken pox
e) Long lasting diarrhea
654. In commonly available antacid preperations, a magnesium salt is mixed with an aluminum salt. Why this
combination is justifiable?
a) This double salt combination potentiates the antacid effects of each other
b) Magnesium salt‟s laxative effect balances the constipation induced by the aluminum salts
c) Magnesium is a mucosal irritant while aluminum is a counter irritant for its effect
d) Magnesium dependant enzyme activates the aluminum salt
e) Magnesium salt being a diuretic prevents the aluminum accumulation

655. A temporary remedy of heart burn from peptic ulcer that can result in stretching of ulcerative lesion
and bleeding from the gastric mucosa:
a) Magnesium trisilicate
b) Aluminum hydroxide
c) Cimetidine
d) Sodium bicarbonate
e) A combination of magnesium and aluminum salts

656. Which one of the follwoign drugs is given to the patient of cirrhosis to prevent the encephalopathy:
a) Lansoprazole
b) Cimetidine
c) Lactulose
d) Loperamide
e) Ketanserin

657. Which one of the following is likely to provide a prompt though temporary relief from the heart burn of
peptic ulceration of stomach mucosa?
a) Lansoprazole
b) Misoprostol
c) Magnesium hydroxide
d) Ranitidine
e) Propantheline

658. In peptic ulcer, antacids are now primarily used for:

a) Prompt pain relief

b) Ulcer he ling

c) Preventing ulcer relapse

d) Control of bleeding from ulcer

e) All of the above

659. Bismuth subsalicylate controls diarrhea by:

a) Adsorption

b) Osmosis

c) Chelation

d) Decreasing GIT motility

e) Exerting its astringent action


660. Kaolin & pectin act by:

a) Chemical neutralization

b) Pro kinetic action

c) Adsorption

d) Slowing intestinal peristalsis

e) Decreasing GIT secretions

661. Out of the following which one agent is an appropriate drug for:
- Gastroesophageal reflux
- Diabetic gastroperesis
- Severe emesis
a) Ketaserin
b) Odansetron
c) Dexamethasone
d) Diphenoxylate
e) Metoclopramide

662. Octreotide is given:

a) Orally

b) Intramuscularly

c) By rectal route

d) To absorb the extra fluid in GIT

e) To decrease GIT motility and secretion

663. The Opioid loperamide:

a) Has a high addiction potential

b) Can not cross the blood brain barrier

c) Controls diarrhea by its physical action

d) Acts through CNS

e) Has no effect on intestinal motility

664. A 64 years old man with a 20 years history of tobacco and alcohol abuse now developed carcinoma of

larynx. His treatment includes high doses of CISPLASTIN and radiation therapy. He developed nausea and

severe vomiting, what would be the best agent to treat these symptoms?

a) Metoclopramide

b) Meclizine

c) Promethazine

d) Ondansetron

e) Glucocorticoids

665. Misoprostol is specially indicated in:


a) Duodenal ulcer caused by H.pylori
b) Stress ulcers of pregnancy
c) Gastroesophageal reflux disease
d) Prevention of long term NSAIDs induced ulcer
e) Prevention of acute stress ulcers of surgery

666. The following drug can completely suppress the gastric acid secretion:
a) Esomeprazole
b) Misoprostal
c) Nizatidine
d) Hyoscine
e) Bismuth subsalicylate

667.A drug that completely suppresses hydrochloric acid secretion in the stomach is:

a) Cimetidine

b) Magnesium hydroxide

c) Lansoprazole

d) Hyoscine compound

e) Misoprostol

668.Which one of the drugs can cause extrapyramidal reaction?

a) Domeridone

b) Methyl dopa

c) Metaclopromide

d) Phenytoin sodium

e) Cimetldine

669. In case of hill journey, antimotion sicknes drugs are best administered at :

a) Twelve hours before commencing journey

b) One hour before commencing journey

c) Immediately after commencing journey

d) At the first feeling of motion sickness

e) 8hrs before start of journey

LIPID LOWERING DRUGS:

670. Recins mainly lower the blood:


a) Triglyceride level

b) Cholestrol

c) HDL

d) VLDL

e) IDL
671.Agents used to treat hypercholesterolemia which work by inhibiting the HMA-CoA reductase enzyme

include:

a) Lovastatin

b) Gemfibrozil

c) Niacin

d) Clofibrate

e) Colestipol

672. Ezitimibe mainly acts at the level of:

a) Liver

b) Faty tissue

c) Endothelium

d) Blood

e) Intestines

673. Statins act by inhibiting :

a) Lipoprotein lipase

b) Amylase

c) HMG COA reductase

d) Aminotransferase

e) Alkaline phosphatase

674. Niacin causes:

a) Bloating and constipation

b) A decrease in HDL

c) An increase in LDL

d) Inhibition in chylomicrons transport

e) Flushing and pruritus

675. Gemfibrozil is a:

a) Recin

b) Phenolic glucoronide

c) HMG coA reductase inhibitor

d) Fibric acid deritive

e) Lipoprotein lipase inhibitor

676. Which of the following should be monitored when statins are used?
a) Renal function tests
b) Ear conduction studies
c) Regular complete blood counts
d) Serum transaminase level
e) Fat soluble vitamin status
677. An obese man with severe triglyceridemia, seeks advice about his medications. Which of the following
drugs is most suitable for him to bring his triglyceride level to normal?
a) Colestipol
b) Ezetimibe
c) Cholestyramine
d) Simvastatin
e) Gemfibrozil

678. Which one of the following drugs lower serum cholesterol by inhibiting 3 hydroxy, 3 methyl glutaryl-
coenzyme-A reductase?
a) Niacin
b) Fenofibrate
c) Mevastatin
d) Genfibrozil
e) Cholestyramine

679. A patient has come to the OPD for routine checkup. Investigations revealed a low HDL level. Which of the
following drugs is advisable?
a) Simvastatin
b) Clofibrate
c) Niacin
d) Gemfibrozil
e) Atorvastatin

680. All of the following are secondary causes of hypertriglyceridemia except one. Which one of the following is
the secondary cause of hypercholesterolemia?
a) Anabolic steroids
b) Estrogen
c) Diabetes mellitus
d) Early nephrosis
e) Alcohol consumption

681. Lipoproteins are atherogenic except one which prevents altherosclerosis and that is:
a) Very low density lipoproteins (VLDL)
b) High density lypoproteins (HDL)
c) Chylomicrons
d) Low density lipoproteins (LDL)
e) Intermediate density lipoproteins (IDL)

682. Which of the following is an inactive form of lipid lowering drug. This undergoes activation in the gut:
a) Simvastatin
b) Fluvastatin
c) Mevastatin
d) Pravastatin
e) Atorvastatin
683. A man suffering from gout is advised some drug for his hyperlipidemias. Which one of the following is
contraindicated for him?
a) Simvastatin
b) Lovastatin
c) Fenofibrate
d) Cholestyramine
e) Niacin

684. Which one of the following is associated with the use of gemfibrozil?
a) Hepatitis
b) Hyperuricemia
c) Kidney stones
d) Cholelithiasis
e) Bloating

685. Statins act by inhibiting:

a) Lipoprotein lipase

b) Amylase

c) HMG COA reductase

d) Aminotransferase

e) Alkaline phosphatase

686. Niacin causes:

a) Bloating and constipation

b) A decrease in HDL

c) An increase in LDL

d) Inhibition in chylomicrons transport

e) Flushing and pruritus

687. Gemfibrozil is a :

a) Recin

b) Phenolic glucoronide

c) HMG coA reductase inhibitor

d) Fibric acid deritive

e) Lipoprotein lipase inhibitor

688. Bile salt binding resins:

a) Increase the secretion of bile salts

b) Decrease the secretion of bile salts

c) Should always be given after meals

d) Chemically neutralize the bile

e) Can cause bloating and constipation


689. Which one of the following lipid lowering drugs, carries a risk of fetal malformations?
a) Niacin
b) Clofibrate
c) Colestipol
d) Simvastatin
e) Ezetimibe

COAGULANTS AND ANTI COAGULANTS

690. A patient comes to OPD with epistaxis and petechial hemorrhages at different sites on the skin. He has
been using warfarin for a long period of time. His international normalized ratio (INR) is considerably
prolonged. Which of the following drugs needs to be given alongwith stopping of warfarin?
a) Streptokinase
b) Protamine sulfate
c) Aminocaproic acid
d) Epoetin alfa
e) Phytonadione

691A patient with DVT was put on unfractionated heparin. On day 4, he developed thrombocytopenia, he

still requires I/V anti coagulation. The patient is most likely to be treated with.:

a) Abcixibab

b) Aprotinin

c) Lepirudin

d) Plasminogen

e) Vitamin K

692. The mechanism of action of protamine is:


a) Stimulates the clotting cascade
b) Hydrolysis of heparin
c) Competitive inhibition of heparin
d) Combines with heparin and inactivates it
e) It changes the conformation of antithrombin III so that heparin cannot bind to it
693.Blood level of which clotting factor declines most rapidly after the inhibition of warfarin therapy

a) Factor VII

b) Factor IX

c) Factor X

d) Prothrombin

e) Factor VIII

694. What might be an indication for administration of a general growth factor like stem cell factor?
a) Metastatic breast cancer
b) Pancytopenia
c) Agranulocytosis
d) Acute myelogenous leukemia
e) Red cell aplasia
695. What is mechanism of action of erythropoietin?
a) It enhances oxygen dissociation from RBCs under hypoxic conditions
b) It promotes the oxygen carrying capacity of myloid precursors
c) It promotes the differentiation of reticulocytes to erythrocytes
d) It promotes the differentiation of myloid cells to reticulocytes
e) It decreases the oxygen binding capacity of hypoxia inducible factor 1α
696. Streptokinase acts by the following mechanism:
a) It activates the conversion of plasminogen to plasmin
b) It prevents the conversion of fibrinogen to fibrin
c) It activates the breakdown of fibrin
d) It inhibits the convertion of prothrombin to thrombin
e) It inhibits the platelets adhesiveness.

697.A 35 year post-partum lady developed DVT. She was admitted in hospital. She was put on heparin,

regarding heparin what is true:

a) Half-life is not dose dependent

b) Can be both orally and IV

c) Can cause thrombocytopenia

d) Activates factors II, XII, IX, X

e) Decrease bleeding

698. A patient with the history of transient ischemic attacks has severe aspirin allergy. Which one of the
following should be advised for him?
a) Streptokinase
b) Clopidogrel
c) Dipyridamole
d) Acetaminophen
e) Aminocaproic acid

699.A 60 years business man with mild hypertension, experiences severe chest pain during exercise. In

emergency, ECG was done which shows ST elevation in anterior leads showing anterior wall myocardial

infarction. Which one will be the best treatment for this patient?

a) Heparin

b) Leperudin

c) Warfarin

d) Tranxenic acid

e) Streptokinase

700. The following is the mechanism of action of clopidogrel:


a) Inhibition of Vit K dependant clotting
b) Activation of plasminogen to plasmin
c) Antagonism of ADP receptor
d) Antagonism of glycoprotein IIb/IIIa
e) Inhibition of platelet thromboxane synthesis
701. A 45 years old lady is given streptokinase for M.I. After the first week it is replaced by alteplase. The
mechanism of action of alteplase is:
a) Inhibition of Vit K dependant clotting factors
b) Activation of plasminogen to plasmin
c) Inhibition of platelet thromboxane synthesis
d) Antagonism of ADP receptor
e) Antagonism of glycoprotein IIb/IIIa
702. The mechanism of action of heparin is:
a) It binds Ca++ in some clotting factors
b) It inhibits thrombin and the early steps of clotting
c) It inhibits the platelet aggregation
d) It increases the plasma level of factor IX
e) It activates plasminogen
703. Warfarin acts by:
a) Inhibition of thrombin
b) Inhibition of platelet aggregation
c) Increases plasma level of factor XI
d) Inhibition of prothrombin synthesis and factors VII, IX and X
e) Activation of plasminogen
704. A patient receiving I/V heparin for one week, had an attack of hematemesis showing excess of heparin.
Which of the following drugs should be given to treat excess of heparin?
a) Vitamin K
b) Ca gluconate
c) Aminocaproic acid
d) Dipyridamole
e) Protamine
705. Which one of the following drugs will inactivate thrombin within the fibrin clot?
a) Dipyridamole
b) Lepirudin
c) Dalteparin
d) Urokinase
e) Danaproid

706. A 55 year old man with atrial fibrillation is taking warfarin. Regarding warfarin what is true?

a) Act by inactivating factor II, VII,IX, X

b) Half-life is decreased by concomitant use of amiodarone

c) It does not crosses the placenta

d) Risk of bleeding is increased

e) All of them

707. Strepto kinase was infused in patient for the management of deep vein thrombosis, following which the

patient developed hematemesis. Which of the given agent can be chosen to manage this episode of

hematemesis?

a) Vitamin K

b) Noradrenaline
c) Epsilon amino caproic acid

d) Rutin

e) Vitamin D

708. The thrombolytic agents:


a) Interfere with the activation of coagulation cascade
b) Interfere with the activation of thrombin
c) Are of clinical value when given with dipyridamole
d) Can reduce inflammation
e) Activate plasmin
709. A chronic alcoholic is brought to the doctor with confusion, nystagmus and bizarre ocular movements.
The doctor after taking the history finds that the patient has been totally ignorant of his nutritional
needs. He should advise which one of the following?
a) Cyanocobalamine
b) Vitamin A and D
c) Folic acid
d) Vitamin C
e) Vitamin B1

710.Erythropoietin is used in which of the following conditions ?

a) Anemia in chronic failure

b) Chronic inflammation

c) AIDS

d) Cancer

e) All

ENDOCRINOLOGY:

2) 710-1 Many factors or drugs are known to affect growth hormone secretion. Those drugs that

stimulate release of growth hormone in normal subjects include all of the following EXCEPT:

a. Clonidine

b. Bromocriptine

c. Growth hormone releasing hormone

d. Serotonin

e. Somatostatin

711. Iodine is transported into thyroid gland by:


a) Halogen co transporter
b) Na- pump
c) K- pump
d) Ca-pump
e) Transport protein

712. The enzyme involved in oxidation of Iodides into in to Iodine is:


a) Oxidoreductase
b) Transaminase
c) Liase
d) Ligase
e) Per oxidase

713. One mode of Propyl thio uracils action is :


a) Destroying thyroid parenchyma
b) Decreasing the level of blood iodine
c) Competitively inhibiting iodide transporter
d) Inhibiting per oxidase enzyme
e) Inhibiting release of thyroid hormone

714. Iodinated contrast media act by:


a) Iodinating thyroglobulin molecule
b) Deiodinating thyroglobulin molecule
c) Inhibiting organification
d) Inhibiting release of thyroid hormone
e) Destroying the thyroid tissue

715. Radio active Iodine acts by destroying the thyroid parenchyma by:
a) α-rays
b) β- rays
c) γ-rays
d) Infra red rays
e) U.V- rays
716. A post menopausal women is using propylthiouracil for her hyperthyroidism. What is the mechanism of
action of this drug?
a) It washes away T3 and T4 from the glandular tissue and releases these into the blood
b) Increases the hepatic breakdown of T3 and T4
c) It blocks iodide trapping into the gland
d) It inhibits the proteolysis of thyroglobulin
e) It inhibits the peroxidase enzyme
717. What is the main objective of giving propranolol in hyperthyroidism?
a) It can lower T3 and T4 levels
b) It overcomes the secondary signs and symptoms
c) It blocks thyroid harmone receptors
d) It inhibits thyroxin release by the thyroid gland
e) It inhibits the synthesis of thyroid harmone

718. A patient with carcinoma thyroid is given oral radioactive iodine (/131). Which statement regarding this
case is correct?
a) Oral antithyroid drugs are also given concomitantly until complete recovery
b) In majority of the patients delayed hypothyroidism results
c) Non thyroid cancers may also develop due to the radiations
d) Once /131 is started, β-blockers should not be used any more
e) 6 – 12 hours after starting radiations, the hyperthyroidism symptoms subside.

719. A patient came to endocrinology OPD in the 2nd trimester with hyperthyroidism. She was put on drug

which blocks the oxidation and iodination of thyroglobulin .Which drug can it be?
a) Methamazole

b) Propyl thiouraicil

c) Lugol‟s iodine

d) Propranolol

e) Corticosteroid

720. For the type I diabetes insulin glargine is distinct from all other insulins in the following respect:
a) It sensitizes the cells to insulin
b) Blood sugar level is maintained at a low continued plateau level
c) Fasting does not cause hypoglycemia at all
d) It exhibits a disulfiram like reaction
e) Being a rapid acting insulin, it has an excellent postprandial blood sugar control

721. During a pituitary radiational treatment for the cushing syndrome which one of the following drug can
effectively inhibit the synthesis of glucocorticoids till the treatment is complete?
a) Ketoconazole
b) Spironolactone
c) Cortisone
d) Cortisol
e) Fludrocortisone
722. Which of the following agent has both glucocorticoid and mineralocorticoid activity?
a) Aldosterone
b) Mifepristone
c) Dexamethasone
d) Fludrocortisone
e) Prednisone
723. Secondary adrenal insufficiency is treated by replacement with:
a) FtuticasoneH
b) Hydrocortisone
c) Fludrocortisone
d) Spironolactone
e) Ketoconazole
724. A 26 years old asthmatic girl using glucocorticoids also starts using oral contraceptives. What interaction
can be anticipated?
a) Antagonism of glucocorticoids and worsening of asthma
b) Contraceptive failure
c) Hypertensive crisis
d) Addisonian crisis
e) Corticosteroids adverse reactions increased

725. Which of the following glucocorticoids produces the least sodium retention?

a) Corticosone

b) Hydrocortieosone

c) Prednisolone

d) Dextramethasone

e) Fludrocortisones
726. An epileptic women using phenytoin sodium, starts oral contraceptives. What can be anticipated about
this women?
a) The efficacy of contraceptive is reduced
b) Thromboembolism
c) Aggranulocytosis due to added toxicity
d) Increased frequency of seizures due to increased elimination of phenytoin
e) Phenytoin toxicity

727. Which one of the following drugs for type 2 diabetes may cause abdominal cramps, grumbling sounds or
flatus?
a) Ciglitazone
b) Acarbose
c) Glimepiride
d) Phenformin
e) Acetohexamide
728. Tolbutamide act by:
(a) Decreasing the absorption of Glucose in G.I.T
(b) Decreasing the utilization of Gucose in the body
(c) Increasing Insulin secretion
(d) Decreasing Insulin secretion
(e) Decreasing Glucagon secretion

729. Glyburide and Glipizide are:


(a) First generation Sulfonylureas
(b) 2nd generation Sulfonylureas
(c) Sulfonylureas with more adverse effects than Tolbutamide
(d) Given by parenteral route
(e) Not given due to lactic acid acidosis

730. Metformin :
(a) Needs functional β cells for its action
(b) Is a sulfonylurea
(c) Is used for type-1 diabetes
(d) Does not need functional β cells for its action
(e) Causes severe hypoglycemia

731. Pioglitazone :
(a) Causes severe lactic acidosis
(b) Acts on its receptors present on the surface of pancreas
(c) Is used for insulin dependant diabetes
(d) Decreases the release of Glucagon
(e) Acts on its own receptors on adipose tissues, hepatocytes and myocytes

732. Acarbose:
(a) Increases Insulin secretion
(b) Decreases glucagon secretion
(c) Increases tissue uptake of Glucose
(d) Competitively inhibits α-glucosidase in G.I.T
(e) Increases tissue utilization of glucose

733. Which of the statements regarding metformin use in type 2 diabetes mellitus is correct?
A. It is an equally effective drug for type I diabetes mellitus
B. Its action is not dependant on the liver function
C. It leads to obesity
D. Lactic acidosis is common but is not of any serious concern
E. Hypoglycemia is very rare
734. Which of the following agents increases secretion of insulin?
a) Pioglitazone
b) Rosiglitazone
c) Meglitinides
d) NPH insulin
e) Miglitol
735. Anorexia, nausea, vomiting, abdominal pain, bloating, difficulty in breathing, liver enlargement, high
blood lactate levels and deteriorating renal function is likely to be associated with:
a) Troglitazone
b) Acarbose
c) Metformin
d) Glyburide
e) Glipizide

736. A drug englitazone given for type 2 diabetes mellitus acts by:
a) Increasing the cells sensitivity to insulin
b) Decreasing the carbohydrate absorption through the gut
c) Increasing insulin secretion
d) Increasing glucose excretion in the urine
e) Increasing gluconeogenesis

737. In type 2 diabetes mellitus, chlorpropamide acts by which one of the following mechanisms: -
a) More insulin secretion from the β cells of islets
b) Decreasing the insulin resistance
c) Increases glucose uptake by liver and muscle cells by insulin dependent process
d) Increasing excretion of glucose by the kidneys
e) Increases the synthesis of insulin

738. A young man suffering from hypercortisolism has to undergo investigation for determining the origin of
the increased ACTH production, whether pituitary or adrenal. For diagnostic cortisone suppression,
which one of the following should be given to him?
a) Hydrocortisone
b) Dexamethasone
c) Prednisone
d) Triamcinolone
e) Methyl prednisone

739. Which one of the following choices for oral contraceptives will have minimal risk of thromboembolism
with good efficacy in a vulnerable patient?
a) Estrogen and progesterone with smaller progestin
b) Estrogen only
c) An estrogen progesterone with small estrogen dose
d) An estrogen progesterone combination with higher progestin
e) An estrogen progesterone combination with higher estrogen

740. Due to the use of a high estrogen combination oral contraceptive, a women can expect which one of the
following adverse effects more likely?
a) Hypertension
b) Weight gain
c) Hypomenorrhea
d) Increased appetite
e) Fatigue

741. Which one of the following mechanisms plays a major role in the contraceptive effect of oral
contraceptives?
a) Reducing uterine blood flow to block the oxygen supply to zygote
b) Acidifying cervical mucus
c) Dislodging fertilized ovum from the endometrial bed
d) Blocking implantation of the fertilized ovum
e) Inhibiting ovulation
742. Estrogen replacement therapy is associated with development of:
a) Coronary heart disease
b) Thromboembolic disease
c) Ovarian cancer
d) Uterine atrophy
e) Endometriosis
743. A patient develops skeletal muscle tetany due to surgical loss of parathyroids what is the strategy that
can provide prompt relief initially:
a) Injection of parathyroid harmone for replacement
b) Calcitonin administration
c) Vit D injection
d) Calcium gluconate intravenously
e) Mithramycin administration

744. A young man already under the treatment of his physician for some medical problems was given
sildenafil by his chemist friend to improve his erectile dysfunction. Shortly after taking the pill, he got
severe hypotension. He was brought to the hospital with tachycardia. What other medicine out of the
following was already in his use?
a) Atenolol
b) Glimepiride
c) Nitroglycerine
d) Digoxin
e) Testosterone

745. A women with primary amenorrhea was prescribed bromocriptine by her physician. After one month of
treatment she established a normal menstrual cycle. What was the mechanism of action of this drug?
a) It stimulated the release of FSH
b) It stimulated GnRH release
c) It caused blocking of estrogen receptor to increase the gonadotropin release
d) It caused inhibition of prolactin release
e) It stimulated estrogen and progestin synthesis
746. A progestin only contraceptive pill differs from a combination pill in that:
a) It causes more menstrual irregularities
b) It is likely to cause thromboembolism
c) It has spermicidal action
d) It is more efficacious
e) It must be taken in a cyclic schedule and not continuously throughout the month

747. Physical combinations or recombinant DNA modifications of insulin are primarily meant to:
a) Prevent K+ uptake as glucose enters the cells
b) Prevent the insulin allergy
c) Stimulate the endogenous insulin synthesis
d) Modify the onset and duration of action
e) Render it suitable for subcutaneous or intravenous use

748. Which of the following agent will block glucocorticoid receptors ?

a) Beclomethasone

b) Ketoconazole

c) Mifepristone

d) Spironolactone

e) Misoprostol

749. An old man of 75 years with frequency and urgency of micturition and with enlarged smooth surfaced
prostate is diagnosed as BPH. He responds well to finasteride. What is the mechanism of action of this
drug?
a) It inhibits the synthesis of testosterone
b) It lowers the serum levels of testosterone by enhancing its metabolism
c) It blocks the synthesis of catecholamines
d) It blocks the testosterone receptors
e) It inhibits 5α-reductase enzyme

750. A 60 years old lady complains of increased thirst and frequency of micturition. Investigation reveals
elevated serum creatinine and CT scan suggests nephrocalcinosis.
She had been using different drugs on the advice of her brother who is a chemist and druggist. Which
drug might have caused her problem?
a) Vitamin D
b) Estrogen
c) Prednisolone
d) Etidronate
e) Glimepiride
751. Which of the following is an estrogen that is used in most combined hormonal contraceptives?

a) Clomiphene

b) Estrone
c) Ethenyl estradiol

d) Drethylstibestrol

e) Nargestrel

3) 751-1. Gonadotropins can be used for the treatment of infertility due to pituitary insufficiency. Side

effects of gonadotropin administration include.

a. Ovarian atrophy and menopausal symptoms

b. Multiple births

c. Hirsutism

d. Ovarian enlargement and rupture

e. Adrenal atrophy

752. Which agent would be appropriate for the management of postmenopausal osteoporotic females?

a) Vitamin D

b) Raloxifene

c) Propranolol

d) Corticosteroids

e) Aspirin

753. Which one of the following is likely to be the most effective for a patient of Vit D resistant rickets:
a) Prednisone
b) Chlorothiazide
c) Calcium gluconate
d) Estrogen
e) Calcitriol
754. A women fears premature labour and her doctor is also concerned about the fetal respiratory distress
syndrome in new born. Which one of the following should be given to prevent respiratory distress
syndrome of the fetus?
a) Betamethasone
b) Indomethacine
c) Propranolol
d) Salbutamol
e) Ergometrine

755. Prophylactic use of tamoxifen for the carcinoma breast:


a) May increase the chances of osteoporosis
b) Prevents the thromboembolic disorders
c) Blocks the estrogen receptors in the breast
d) Prevents endometrial carcinoma
e) Increases the blood cholesterol

756. Which one of the following drugs can act as post coital contraceptive?
a) Mifepristone
b) Ergometrine
c) Tamoxifen
d) Raloxefene
e) Ritodrine

757.Which of the following drug is used as an emergency contraceptive?

a) Clomiphene

b) Danazol

c) Ergometrine

d) Levonorgestrel

e) Toremifene

758. Which one of the following adverse effects is likely to be associated with chlorpropamide?
a) Hyponatremia
b) Disulfiram like reaction with alcohol consumption
c) Hyperosmolar urine
d) Weight gain
e) Symptomatic hypothyroidism

759. The following effects are common with the persistant anabolic steroid use (abuse) except:
a) Fluid retention
b) Decreased spermatogenesis
c) Masculinization of females
d) Psychosis or depression
e) Loss of appetite
760. Osteoporosis is the major adverse effect caused by the glucocorticoid. Its due to their ability to :

a) Increase excretion of calcium

b) Inhibit absorption of calcium

c) Stimulate the hypothalamic pituitary adrenal axis

d) Stimulate the adrenal cortex

e) Decrease the production of prostaglandins

761. A patient using danazol for endometriosis should be warned about which one of the following for regular
monitoring?
a) Weight loss
b) Anemia
c) Psychosis
d) Liver dysfunction
e) Thrombocytopenia
762. Which one of the following is the effect of parathyroid harmone?
a) Decreasing reabsorption of phosphate from the bones
b) Increasing mobilization of Ca++ from the bones
c) Decreasing the calcium absorption in the intestine by active transport
d) Decreasing the excretion of phosphate
e) Decreasing calcium reabsorption by the renal tubules
763. A hyperthyroid patient has been advised to take methimazole. Which one of the following serious adverse
effects associated with this drug requires regular monitoring:
a) Malignant change
b) Hyperuricemia
c) Agranulocytosis
d) Hepatitis
e) Renal failure

764. Which of the following endocrine disorders is associated with amiodarone used in arrhythmia?
a) Cushing syndrome
b) Addisons disease
c) Diabetes mellitus
d) Hypothyroidism
e) Diabetes insipidus

765. Hypoglycemia cannot be treated rapidly by taking sugar if the combination of antidiabetic drugs for type
2 diabetes includes:
a) Pioglitazone
b) Repaglinide
c) Glyburide
d) Metformin
e) Acarbose

766. The insulin preparation of choice in diabetic ketoacidosis is :

a) Regular insulin

b) Lente insulin

c) Isophane isulin

d) Monocomponent insulin

767. Which of the following endocrine functions is tested by using metyrapone?


a) Thyroid response to TSH
b) Pituitary adrenal axis
c) α cells of islands of langerhans
d) β cells of islands of langerhans
e) Cells of sertoli
768. Which oral antidiabetic agent of the following is most likely to aggravate a day long fasting
hypoglycemia?
a) Ciglitazone
b) Englitazone
c) Acarbose
d) Glyburide
e) Metformin
769. Which of the following drugs is taken during the first part of a meal for the purpose of delaying the

absorption of dietary carbohydrates?

a) Acarbose

b) Exenatide

c) Glipizide

d) Pioglitazone

e) Repaglimide
770.Glimepiride, the second generation sulfonylurea, acts by increasing the release of insulin by :

a) Activating Ca channels

b) Blocking Ca channels

c) Blocking K channels

d) Blocking Na channels

e) Inhibiting insulin receptor

771. Which one of the following drugs is most appropriate for suppressing the post operative metastasis of
carcinoma of prostate?
a) Mifepristone
b) Aminoglutethimide
c) Spironolactone
d) Fludrocortisone
e) Leuprolide

You might also like